Slot System
Featured Buckets
Featured Buckets Admin

Are we causing anemia by ordering unnecessary blood tests?

Article Type
Changed
Wed, 08/16/2017 - 14:25
Display Headline
Are we causing anemia by ordering unnecessary blood tests?

A 68-year-old woman is admitted for community-acquired pneumonia. She receives antibiotics, and her condition begins to improve after 2 days. She has her blood drawn daily throughout her admission.

On hospital day 3, she complains of fatigue, and on day 4, laboratory results show that her hemoglobin and hematocrit values have fallen. To make sure this result is not spurious, her blood is drawn again to repeat the test. On day 5, her hemoglobin level has dropped to 7.0 g/dL, which is 2 g/dL lower than at admission, and she receives a transfusion.

On day 7, her hemoglobin level is stable at 8.5 g/dL, and her physicians decide to discharge her. The morning of her discharge, as a nurse is about to draw her blood, the patient asks, “Are all these blood tests really necessary?”

DO WE DRAW TOO MUCH BLOOD?

This case portrays a common occurrence. Significant amounts of blood are drawn from patients, especially in critical care. Clinical uncertainty drives most laboratory testing ordered by physicians. Too often, however, these tests lead to more testing and interventions, without a clear benefit to the patient.1

When blood testing leads to more testing, a patient’s hemoglobin and hematocrit can fall. Symptomatic iatrogenic anemia is associated with significant morbidity for patients with preexisting cardiopulmonary disease.

We draw much larger volumes of blood than most testing guidelines say are necessary. One author2 has noted that 50 to 60 mL of blood is removed for each set of tests, owing to the size of collection tubes, multiple reagents needed for each test, and the possibility that tests may need to be rerun. Yet about 3 mL of blood is sufficient to perform most laboratory tests even if the test needs to be rerun.2

CAN BLOOD DRAWS CAUSE ANEMIA?

A relationship between the volume of blood drawn and iatrogenic anemia was first described in 2005, when Thavendiranathan et al3 found that in adult patients on general medicine floors, the volume of blood drawn strongly predicted decreased hemoglobin and hematocrit levels. For every 100 mL of blood drawn, hemoglobin levels fell by an average of 0.7 g/dL, and 13.9% of the patients in the study had iron studies and fecal occult blood tests performed to investigate anemia.

Kurniali et al4 reported that during an average admission, 65% of patients experienced a drop in hemoglobin of 1.0 g/dL or more, and 49% developed anemia.

Iatrogenic anemia from blood draws is common, serious, and unnecessary

Salisbury et al,5 in 2011, studied 17,676 patients with acute myocardial infarction across 57 centers and found a correlation between the volume of blood taken and the development of anemia. On average, for every 50 mL of blood drawn, the risk of moderate to severe iatrogenic anemia increased by 18%. They also found significant variation in blood loss from testing in patients who developed moderate or severe anemia. The authors believed this indicated that moderate to severe anemia was more frequent at centers with higher than average diagnostic blood loss.5

This relationship has also been described in patients in intensive care, where it contributes to anemia of chronic disease. While anemia of critical illness is multifactorial, phlebotomy contributes to anemia in both short- and long-term stays in the intensive care unit.6

CHOOSING WISELY GUIDELINES

The Choosing Wisely initiative of the American Board of Internal Medicine Foundation collects recommendations by a number of medical specialty societies to reduce overuse of healthcare resources.7 The Critical Care Societies Collaborative recommends ordering diagnostic tests only when they answer specific clinical questions rather than routinely. The Society of Hospital Medicine also recommends against repeat complete blood cell count and blood chemistry testing because it may contribute to anemia, which is of particular concern in patients with cardiorespiratory disease.

POSSIBLE HARM

The Critical Care Societies Collaborative, in its Choosing Wisely Guidelines, specifically cites anemia as a potential harm of unnecessary phlebotomy, noting it may result in transfusion, with its associated risks and costs. In addition, aggressive investigation of incidental and nonpathologic results of routine studies is wasteful and exposes the patient to additional risks.

REDUCING PHLEBOTOMY DECREASES IATROGENIC ANEMIA

Since the relationship between excessive phlebotomy and iatrogenic anemia was described, hospitals have attempted to address the problem.

Order tests only to answer specific clinical questions—not routinely

In 2011, Stuebing and Miner8 described an intervention in which the house staff and attending physicians on non-intensive care surgical services were given weekly reports of the cost of the laboratory services for the previous week. They found that simply making providers aware of the cost of their tests reduced the number of tests ordered and resulted in significant hospital savings.

Another strategy is to use pediatric collection tubes in adult patients. A 2008 study in which all blood samples were drawn using pediatric tubes reduced the blood volume removed per patient by almost 75% in inpatient and critical care patients, without the need for repeat blood draws.9 However, Kurniali et al found that the use of pediatric collection tubes did not significantly change hemoglobin fluctuations throughout patient hospital stays.4

Corson et al10 in 2015 described an intervention involving detailing, auditing, and giving feedback regarding the frequency of laboratory tests commonly ordered by a group of hospitalists. The intervention resulted in a modest reduction in the number of common laboratory tests ordered per patient day and in hospital costs, without any changes in the length of hospital stay, mortality rate, or readmission rate.10

THE CLINICAL BOTTOM LINE

As a general principle, diagnostic testing should be done to answer specific diagnostic questions and to guide management. Ordering of diagnostic tests should be decided on a day-to-day basis rather than scheduled automatically or done reflexively. In the case of blood draws, the volume of blood drawn is significantly increased by unnecessary testing, resulting in higher rates of hospital-acquired anemia.

References
  1. Ezzie ME, Aberegg SK, O’Brien JM Jr. Laboratory testing in the intensive care unit. Crit Care Clin 2007; 23:435–465.
  2. Stefanini M. Iatrogenic anemia (can it be prevented?). J Thromb Haemost 2014; 12:1591.
  3. Thavendiranathan P, Bagai A, Ebidia A, Detsky AS, Choudhry NK. Do blood tests cause anemia in hospitalized patients? The effect of diagnostic phlebotomy on hemoglobin and hematocrit levels. J Gen Intern Med 2005; 20:520–524.
  4. Kurniali PC, Curry S, Brennan KW, et al. A retrospective study investigating the incidence and predisposing factors of hospital-acquired anemia. Anemia 2014; 2014:634582.
  5. Salisbury AC, Reid KJ, Alexander KP, et al. Diagnostic blood loss from phlebotomy and hospital-acquired anemia during acute myocardial infarction. Arch Intern Med 2011; 171:1646–1653.
  6. Walsh TS, Lee RJ, Maciver CR, et al. Anemia during and at discharge from intensive care: the impact of restrictive blood transfusion practice. Intensive Care Med 2006; 32:100–109.
  7. American Board of Internal Medicine Foundation. Choosing Wisely. www.abimfoundation.org/Initiatives/Choosing-Wisely.aspx. Accessed April 19, 2016.
  8. Stuebing EA, Miner TJ. Surgical vampires and rising health care expenditure: reducing the cost of daily phlebotomy. Arch Surg 2011; 146:524–527.
  9. Sanchez-Giron F, Alvarez-Mora F. Reduction of blood loss from laboratory testing in hospitalized adult patients using small-volume (pediatric) tubes. Arch Pathol Lab Med 2008; 132:1916–1919.
  10. Corson AH, Fan VS, White T, et al. A multifaceted hospitalist quality improvement intervention: decreased frequency of common labs. J Hosp Med 2015; 10:390–395.
Article PDF
Author and Disclosure Information

Carlo Lutz, MD
Department of Emergency Medicine, Jacobi Medical Center, Bronx, NY

Hyung J. Cho, MD
Director, Quality and Patient Safety, Division of Hospital Medicine; Assistant Professor of Medicine, Icahn School of Medicine at Mount Sinai, New York, NY

Address: Carlo Lutz, MD, Jacobi Medical Center, Department of Emergency Medicine, 1400 Pelham Parkway South, Bronx, NY 10461; carlo.lutz.jacobi.em@gmail.com

Issue
Cleveland Clinic Journal of Medicine - 83(7)
Publications
Topics
Page Number
496-497
Legacy Keywords
Anemia, blood tests, iatrogenic, phlebotomy, blood draw, smart testing, Choosing Wisely, Carlo Lutz, Hyung Cho
Sections
Author and Disclosure Information

Carlo Lutz, MD
Department of Emergency Medicine, Jacobi Medical Center, Bronx, NY

Hyung J. Cho, MD
Director, Quality and Patient Safety, Division of Hospital Medicine; Assistant Professor of Medicine, Icahn School of Medicine at Mount Sinai, New York, NY

Address: Carlo Lutz, MD, Jacobi Medical Center, Department of Emergency Medicine, 1400 Pelham Parkway South, Bronx, NY 10461; carlo.lutz.jacobi.em@gmail.com

Author and Disclosure Information

Carlo Lutz, MD
Department of Emergency Medicine, Jacobi Medical Center, Bronx, NY

Hyung J. Cho, MD
Director, Quality and Patient Safety, Division of Hospital Medicine; Assistant Professor of Medicine, Icahn School of Medicine at Mount Sinai, New York, NY

Address: Carlo Lutz, MD, Jacobi Medical Center, Department of Emergency Medicine, 1400 Pelham Parkway South, Bronx, NY 10461; carlo.lutz.jacobi.em@gmail.com

Article PDF
Article PDF
Related Articles

A 68-year-old woman is admitted for community-acquired pneumonia. She receives antibiotics, and her condition begins to improve after 2 days. She has her blood drawn daily throughout her admission.

On hospital day 3, she complains of fatigue, and on day 4, laboratory results show that her hemoglobin and hematocrit values have fallen. To make sure this result is not spurious, her blood is drawn again to repeat the test. On day 5, her hemoglobin level has dropped to 7.0 g/dL, which is 2 g/dL lower than at admission, and she receives a transfusion.

On day 7, her hemoglobin level is stable at 8.5 g/dL, and her physicians decide to discharge her. The morning of her discharge, as a nurse is about to draw her blood, the patient asks, “Are all these blood tests really necessary?”

DO WE DRAW TOO MUCH BLOOD?

This case portrays a common occurrence. Significant amounts of blood are drawn from patients, especially in critical care. Clinical uncertainty drives most laboratory testing ordered by physicians. Too often, however, these tests lead to more testing and interventions, without a clear benefit to the patient.1

When blood testing leads to more testing, a patient’s hemoglobin and hematocrit can fall. Symptomatic iatrogenic anemia is associated with significant morbidity for patients with preexisting cardiopulmonary disease.

We draw much larger volumes of blood than most testing guidelines say are necessary. One author2 has noted that 50 to 60 mL of blood is removed for each set of tests, owing to the size of collection tubes, multiple reagents needed for each test, and the possibility that tests may need to be rerun. Yet about 3 mL of blood is sufficient to perform most laboratory tests even if the test needs to be rerun.2

CAN BLOOD DRAWS CAUSE ANEMIA?

A relationship between the volume of blood drawn and iatrogenic anemia was first described in 2005, when Thavendiranathan et al3 found that in adult patients on general medicine floors, the volume of blood drawn strongly predicted decreased hemoglobin and hematocrit levels. For every 100 mL of blood drawn, hemoglobin levels fell by an average of 0.7 g/dL, and 13.9% of the patients in the study had iron studies and fecal occult blood tests performed to investigate anemia.

Kurniali et al4 reported that during an average admission, 65% of patients experienced a drop in hemoglobin of 1.0 g/dL or more, and 49% developed anemia.

Iatrogenic anemia from blood draws is common, serious, and unnecessary

Salisbury et al,5 in 2011, studied 17,676 patients with acute myocardial infarction across 57 centers and found a correlation between the volume of blood taken and the development of anemia. On average, for every 50 mL of blood drawn, the risk of moderate to severe iatrogenic anemia increased by 18%. They also found significant variation in blood loss from testing in patients who developed moderate or severe anemia. The authors believed this indicated that moderate to severe anemia was more frequent at centers with higher than average diagnostic blood loss.5

This relationship has also been described in patients in intensive care, where it contributes to anemia of chronic disease. While anemia of critical illness is multifactorial, phlebotomy contributes to anemia in both short- and long-term stays in the intensive care unit.6

CHOOSING WISELY GUIDELINES

The Choosing Wisely initiative of the American Board of Internal Medicine Foundation collects recommendations by a number of medical specialty societies to reduce overuse of healthcare resources.7 The Critical Care Societies Collaborative recommends ordering diagnostic tests only when they answer specific clinical questions rather than routinely. The Society of Hospital Medicine also recommends against repeat complete blood cell count and blood chemistry testing because it may contribute to anemia, which is of particular concern in patients with cardiorespiratory disease.

POSSIBLE HARM

The Critical Care Societies Collaborative, in its Choosing Wisely Guidelines, specifically cites anemia as a potential harm of unnecessary phlebotomy, noting it may result in transfusion, with its associated risks and costs. In addition, aggressive investigation of incidental and nonpathologic results of routine studies is wasteful and exposes the patient to additional risks.

REDUCING PHLEBOTOMY DECREASES IATROGENIC ANEMIA

Since the relationship between excessive phlebotomy and iatrogenic anemia was described, hospitals have attempted to address the problem.

Order tests only to answer specific clinical questions—not routinely

In 2011, Stuebing and Miner8 described an intervention in which the house staff and attending physicians on non-intensive care surgical services were given weekly reports of the cost of the laboratory services for the previous week. They found that simply making providers aware of the cost of their tests reduced the number of tests ordered and resulted in significant hospital savings.

Another strategy is to use pediatric collection tubes in adult patients. A 2008 study in which all blood samples were drawn using pediatric tubes reduced the blood volume removed per patient by almost 75% in inpatient and critical care patients, without the need for repeat blood draws.9 However, Kurniali et al found that the use of pediatric collection tubes did not significantly change hemoglobin fluctuations throughout patient hospital stays.4

Corson et al10 in 2015 described an intervention involving detailing, auditing, and giving feedback regarding the frequency of laboratory tests commonly ordered by a group of hospitalists. The intervention resulted in a modest reduction in the number of common laboratory tests ordered per patient day and in hospital costs, without any changes in the length of hospital stay, mortality rate, or readmission rate.10

THE CLINICAL BOTTOM LINE

As a general principle, diagnostic testing should be done to answer specific diagnostic questions and to guide management. Ordering of diagnostic tests should be decided on a day-to-day basis rather than scheduled automatically or done reflexively. In the case of blood draws, the volume of blood drawn is significantly increased by unnecessary testing, resulting in higher rates of hospital-acquired anemia.

A 68-year-old woman is admitted for community-acquired pneumonia. She receives antibiotics, and her condition begins to improve after 2 days. She has her blood drawn daily throughout her admission.

On hospital day 3, she complains of fatigue, and on day 4, laboratory results show that her hemoglobin and hematocrit values have fallen. To make sure this result is not spurious, her blood is drawn again to repeat the test. On day 5, her hemoglobin level has dropped to 7.0 g/dL, which is 2 g/dL lower than at admission, and she receives a transfusion.

On day 7, her hemoglobin level is stable at 8.5 g/dL, and her physicians decide to discharge her. The morning of her discharge, as a nurse is about to draw her blood, the patient asks, “Are all these blood tests really necessary?”

DO WE DRAW TOO MUCH BLOOD?

This case portrays a common occurrence. Significant amounts of blood are drawn from patients, especially in critical care. Clinical uncertainty drives most laboratory testing ordered by physicians. Too often, however, these tests lead to more testing and interventions, without a clear benefit to the patient.1

When blood testing leads to more testing, a patient’s hemoglobin and hematocrit can fall. Symptomatic iatrogenic anemia is associated with significant morbidity for patients with preexisting cardiopulmonary disease.

We draw much larger volumes of blood than most testing guidelines say are necessary. One author2 has noted that 50 to 60 mL of blood is removed for each set of tests, owing to the size of collection tubes, multiple reagents needed for each test, and the possibility that tests may need to be rerun. Yet about 3 mL of blood is sufficient to perform most laboratory tests even if the test needs to be rerun.2

CAN BLOOD DRAWS CAUSE ANEMIA?

A relationship between the volume of blood drawn and iatrogenic anemia was first described in 2005, when Thavendiranathan et al3 found that in adult patients on general medicine floors, the volume of blood drawn strongly predicted decreased hemoglobin and hematocrit levels. For every 100 mL of blood drawn, hemoglobin levels fell by an average of 0.7 g/dL, and 13.9% of the patients in the study had iron studies and fecal occult blood tests performed to investigate anemia.

Kurniali et al4 reported that during an average admission, 65% of patients experienced a drop in hemoglobin of 1.0 g/dL or more, and 49% developed anemia.

Iatrogenic anemia from blood draws is common, serious, and unnecessary

Salisbury et al,5 in 2011, studied 17,676 patients with acute myocardial infarction across 57 centers and found a correlation between the volume of blood taken and the development of anemia. On average, for every 50 mL of blood drawn, the risk of moderate to severe iatrogenic anemia increased by 18%. They also found significant variation in blood loss from testing in patients who developed moderate or severe anemia. The authors believed this indicated that moderate to severe anemia was more frequent at centers with higher than average diagnostic blood loss.5

This relationship has also been described in patients in intensive care, where it contributes to anemia of chronic disease. While anemia of critical illness is multifactorial, phlebotomy contributes to anemia in both short- and long-term stays in the intensive care unit.6

CHOOSING WISELY GUIDELINES

The Choosing Wisely initiative of the American Board of Internal Medicine Foundation collects recommendations by a number of medical specialty societies to reduce overuse of healthcare resources.7 The Critical Care Societies Collaborative recommends ordering diagnostic tests only when they answer specific clinical questions rather than routinely. The Society of Hospital Medicine also recommends against repeat complete blood cell count and blood chemistry testing because it may contribute to anemia, which is of particular concern in patients with cardiorespiratory disease.

POSSIBLE HARM

The Critical Care Societies Collaborative, in its Choosing Wisely Guidelines, specifically cites anemia as a potential harm of unnecessary phlebotomy, noting it may result in transfusion, with its associated risks and costs. In addition, aggressive investigation of incidental and nonpathologic results of routine studies is wasteful and exposes the patient to additional risks.

REDUCING PHLEBOTOMY DECREASES IATROGENIC ANEMIA

Since the relationship between excessive phlebotomy and iatrogenic anemia was described, hospitals have attempted to address the problem.

Order tests only to answer specific clinical questions—not routinely

In 2011, Stuebing and Miner8 described an intervention in which the house staff and attending physicians on non-intensive care surgical services were given weekly reports of the cost of the laboratory services for the previous week. They found that simply making providers aware of the cost of their tests reduced the number of tests ordered and resulted in significant hospital savings.

Another strategy is to use pediatric collection tubes in adult patients. A 2008 study in which all blood samples were drawn using pediatric tubes reduced the blood volume removed per patient by almost 75% in inpatient and critical care patients, without the need for repeat blood draws.9 However, Kurniali et al found that the use of pediatric collection tubes did not significantly change hemoglobin fluctuations throughout patient hospital stays.4

Corson et al10 in 2015 described an intervention involving detailing, auditing, and giving feedback regarding the frequency of laboratory tests commonly ordered by a group of hospitalists. The intervention resulted in a modest reduction in the number of common laboratory tests ordered per patient day and in hospital costs, without any changes in the length of hospital stay, mortality rate, or readmission rate.10

THE CLINICAL BOTTOM LINE

As a general principle, diagnostic testing should be done to answer specific diagnostic questions and to guide management. Ordering of diagnostic tests should be decided on a day-to-day basis rather than scheduled automatically or done reflexively. In the case of blood draws, the volume of blood drawn is significantly increased by unnecessary testing, resulting in higher rates of hospital-acquired anemia.

References
  1. Ezzie ME, Aberegg SK, O’Brien JM Jr. Laboratory testing in the intensive care unit. Crit Care Clin 2007; 23:435–465.
  2. Stefanini M. Iatrogenic anemia (can it be prevented?). J Thromb Haemost 2014; 12:1591.
  3. Thavendiranathan P, Bagai A, Ebidia A, Detsky AS, Choudhry NK. Do blood tests cause anemia in hospitalized patients? The effect of diagnostic phlebotomy on hemoglobin and hematocrit levels. J Gen Intern Med 2005; 20:520–524.
  4. Kurniali PC, Curry S, Brennan KW, et al. A retrospective study investigating the incidence and predisposing factors of hospital-acquired anemia. Anemia 2014; 2014:634582.
  5. Salisbury AC, Reid KJ, Alexander KP, et al. Diagnostic blood loss from phlebotomy and hospital-acquired anemia during acute myocardial infarction. Arch Intern Med 2011; 171:1646–1653.
  6. Walsh TS, Lee RJ, Maciver CR, et al. Anemia during and at discharge from intensive care: the impact of restrictive blood transfusion practice. Intensive Care Med 2006; 32:100–109.
  7. American Board of Internal Medicine Foundation. Choosing Wisely. www.abimfoundation.org/Initiatives/Choosing-Wisely.aspx. Accessed April 19, 2016.
  8. Stuebing EA, Miner TJ. Surgical vampires and rising health care expenditure: reducing the cost of daily phlebotomy. Arch Surg 2011; 146:524–527.
  9. Sanchez-Giron F, Alvarez-Mora F. Reduction of blood loss from laboratory testing in hospitalized adult patients using small-volume (pediatric) tubes. Arch Pathol Lab Med 2008; 132:1916–1919.
  10. Corson AH, Fan VS, White T, et al. A multifaceted hospitalist quality improvement intervention: decreased frequency of common labs. J Hosp Med 2015; 10:390–395.
References
  1. Ezzie ME, Aberegg SK, O’Brien JM Jr. Laboratory testing in the intensive care unit. Crit Care Clin 2007; 23:435–465.
  2. Stefanini M. Iatrogenic anemia (can it be prevented?). J Thromb Haemost 2014; 12:1591.
  3. Thavendiranathan P, Bagai A, Ebidia A, Detsky AS, Choudhry NK. Do blood tests cause anemia in hospitalized patients? The effect of diagnostic phlebotomy on hemoglobin and hematocrit levels. J Gen Intern Med 2005; 20:520–524.
  4. Kurniali PC, Curry S, Brennan KW, et al. A retrospective study investigating the incidence and predisposing factors of hospital-acquired anemia. Anemia 2014; 2014:634582.
  5. Salisbury AC, Reid KJ, Alexander KP, et al. Diagnostic blood loss from phlebotomy and hospital-acquired anemia during acute myocardial infarction. Arch Intern Med 2011; 171:1646–1653.
  6. Walsh TS, Lee RJ, Maciver CR, et al. Anemia during and at discharge from intensive care: the impact of restrictive blood transfusion practice. Intensive Care Med 2006; 32:100–109.
  7. American Board of Internal Medicine Foundation. Choosing Wisely. www.abimfoundation.org/Initiatives/Choosing-Wisely.aspx. Accessed April 19, 2016.
  8. Stuebing EA, Miner TJ. Surgical vampires and rising health care expenditure: reducing the cost of daily phlebotomy. Arch Surg 2011; 146:524–527.
  9. Sanchez-Giron F, Alvarez-Mora F. Reduction of blood loss from laboratory testing in hospitalized adult patients using small-volume (pediatric) tubes. Arch Pathol Lab Med 2008; 132:1916–1919.
  10. Corson AH, Fan VS, White T, et al. A multifaceted hospitalist quality improvement intervention: decreased frequency of common labs. J Hosp Med 2015; 10:390–395.
Issue
Cleveland Clinic Journal of Medicine - 83(7)
Issue
Cleveland Clinic Journal of Medicine - 83(7)
Page Number
496-497
Page Number
496-497
Publications
Publications
Topics
Article Type
Display Headline
Are we causing anemia by ordering unnecessary blood tests?
Display Headline
Are we causing anemia by ordering unnecessary blood tests?
Legacy Keywords
Anemia, blood tests, iatrogenic, phlebotomy, blood draw, smart testing, Choosing Wisely, Carlo Lutz, Hyung Cho
Legacy Keywords
Anemia, blood tests, iatrogenic, phlebotomy, blood draw, smart testing, Choosing Wisely, Carlo Lutz, Hyung Cho
Sections
Disallow All Ads
Alternative CME
Article PDF Media

Serum allergen-specific IgE testing: How much is too much?

Article Type
Changed
Tue, 08/15/2017 - 14:18
Display Headline
Serum allergen-specific IgE testing: How much is too much?

A 25-year-old man is evaluated for angioedema (swelling of lips and tongue) after eating paella at a Spanish restaurant. He has no history of allergies, but he says he had never eaten such a large variety of seafood before, especially shellfish.

He suspects that he is allergic to shellfish and asks the attending physician to order blood tests for seafood allergies, as he heard from a friend that blood tests are superior to other types of tests for allergy. The physician requests a serum immunoglobulin E (IgE) food panel test for this patient.

SERUM ALLERGEN-SPECIFIC IgE TESTING

Many methods of testing for allergy are available, including the skin-prick test, double-blind and single-blind placebo-controlled food challenges, open food challenges, inhalant challenges, drug challenges, and serum IgE tests. In clinical practice, these tests are often used in combination because when used individually, few of them are both highly sensitive and specific (Table 1).1–6

Skin-prick testing is generally the method of choice for the preliminary evaluation of IgE-mediated allergies because it is more sensitive and requires less time to get a result.1 But it is not the preferred test if the patient is at risk of a systemic reaction or has widespread dermatitis, nor is it useful if the patient is taking drugs that suppress the histamine response, such as antihistamines or tricyclic antidepressants.6 Moreover, skin-prick testing is more invasive and time-consuming than serum IgE testing.

Serum IgE testing is an attractive alternative, and it is more convenient because it requires only a single blood draw and poses a lower risk of adverse effects.

NOT A RELIABLE DIAGNOSTIC TOOL

As serum IgE testing has gained popularity, researchers have tried to improve its diagnostic power (ie, maximize its sensitivity and specificity) by determining the best cutoff values for IgE against specific antigens. Unfortunately, these values are difficult to determine because of confounding factors such as the lack of a reference standard, population diversity, patient atopy, and the overwhelming number of allergens that must be examined.

In addition, some researchers have used positive and negative predictive values to evaluate diagnostic cutoffs for serum antigen-specific IgE values. But these are not the most suitable performance measure to evaluate because they depend on disease prevalence and population characteristics.

Despite these efforts, results are still conflicting, and serum antigen-specific IgE testing is not a reliable diagnostic tool.

Figure 1. Sum of sensitivity and specificity of serum antigen-specific IgE tests of different ImmunoCAP allergens. A sum of 170 or greater (dashed line) is considered clinically relevant; tests with IgE cutoffs greater than 0.35 kU/L are noted with an asterisk.

In an effort to gain insight from the available research data, we evaluated the clinical usefulness of 89 antigen-specific IgE tests, using an approach of summing their sensitivity and specificity. Previously, Wians7 proposed that a test is likely to be clinically useful if the sum of its sensitivity and specificity is equal to or greater than 170. Figure 1 shows the 89 tests, grouped into categories, and their summed sensitivities and specificities. The dashed line indicates a cutoff of 170; any bar that touches or crosses that line indicates that the test may be clinically useful, according to Wians.7

Only 7 of the 89 tests (cow, buckwheat, hazelnut, latex, Alternaria alternata, honey bee venom, and Johnson grass) satisfied this criterion. This suggests that a significant number of serum antigen-specific IgE tests perform suboptimally, and we are left with the question of why they are so commonly ordered.

Inappropriate use can lead to false-positive results, a situation in which patients may be subjected to unnecessary food avoidance that can result in nutritional deficiencies and decreased quality of life. It can also lead to false-negative results, when life-threatening diagnoses are missed and further excessive downstream testing is required—all leading; to negative outcomes for both patients and healthcare providers.

CHOOSING WISELY

The Choosing Wisely campaign in the United States has partnered with the American Academy of Allergy, Asthma, and Immunology to advocate against indiscriminate IgE testing in evaluating allergy.8 Allergy diagnosis and evaluation should be based on a combination of clinical history and judicious ordering of specific IgE tests, whether through skin or blood testing. Ordering of serum allergen-specific IgE tests for food allergies should be consistent with a clinical history of potential IgE-mediated food allergy8 and not food intolerance (Table 2).4,5

Some jurisdictions in Canada have followed suit by restricting the number of serum IgE tests each physician is allowed to order per patient, to encourage more responsible ordering and to lower the number of potential false-positive results, which can lead to increased downstream costs as well as unnecessary patient worry and lifestyle modification.

 

 

CLINICAL BOTTOM LINE

Ordering diagnostic tests that have little clinical utility has long-term detrimental effects on both patient safety and healthcare sustainability.

In the case of the 25-year-old evaluated for shellfish allergy, the clinician should first explain that the swelling of the lips and tongue (angioedema) does suggest an IgE-mediated allergic reaction and not a non–IgE-mediated allergic reaction or a food intolerance. Non–IgE-mediated food allergies and food intolerances are marked by symptoms relating mainly to nonimmune aspects of the digestive system, whereas IgE-mediated food allergies affect the immune system and can involve a multitude of organs, including the skin and the respiratory and digestive systems (Table 2).

However, clinicians should avoid indiscriminately ordering food allergen IgE panels and instead should focus on foods likely to be the culprits based on the clinical history.9 Indiscriminate testing can lead to false-positive results and unnecessary food avoidance.

Since the patient developed symptoms of angioedema when he was exposed to his allergen, he may be apprehensive about a skin- prick test and the possibility of being subjected to the same discomfort. Therefore, in this situation, it may be best to perform serum IgE tests, but on a few targeted seafoods rather than the food panel the physician had ordered. A patient can be sensitized to an allergen (possess IgE antibodies) but not experience symptoms when exposed to it (ie, have tolerance).5 Also, false-negative results may occur, so a negative serum allergen-specific IgE test should likewise be interpreted in light of the pretest probability of allergy to a specific antigen.

If the history and the results of testing are not clear and congruent, the patient should be referred to an allergist for diagnosis or for management. The allergist can provide management techniques and periodic assessment as to the progression and resolution of the allergy. Table 2 highlights symptoms that differentiate an IgE-mediated from a non–IgE-mediated food allergy.10,11Table 1 presents clinical indications and suggested diagnostic methods to the five most common allergen groups and the diagnostically invalid tests.1–6

The bottom line is that we must consider the poor performance of serum allergen-specific IgE tests when diagnosing and treating suspected type I allergies and avoid ordering food allergen IgE panels whenever possible.

References
  1. Bernstein IL, Li JT, Bernstein DI, et al; American Academy of Allergy, Asthma and Immunology; American College of Allergy, Asthma and Immunology. Allergy diagnostic testing: an updated practice parameter. Ann Allergy Asthma Immunol 2008; 100(suppl 3):S1–S148.
  2. Bird JA, Crain M, Varshney P. Food allergen panel testing often results in misdiagnosis of food allergy. J Pediatr 2015; 166:97–100.
  3. Kattan JD, Sicherer SH. Optimizing the diagnosis of food allergy. Immunol Allergy Clin North Am 2015; 35:61–76.
  4. Sampson HA, Aceves S, Bock SA, et al. Food allergy: a practice parameter update-2014. J Allergy Clin Immunol 2014; 134:1016–1025.e43.
  5. Sicherer SH, Sampson HA. Food allergy: epidemiology, pathogenesis, diagnosis, and treatment. J Allergy Clin Immunol 2014; 133:291–308.
  6. Siles RI, Hsieh FH. Allergy blood testing: a practical guide for clinicians. Cleve Clin J Med 2011; 78:585–592.
  7. Wians FH Jr. Clinical laboratory tests: which, why, and what do the results mean? Lab Medicine 2009; 40:105–113.
  8. Choosing Wisely. American Academy of Allergy, Asthma & Immunology. Ten Things Physicians and Patients Should Question. www.choosingwisely.org/doctor-patient-lists/american-academy-of-allergy-asthma-immunology/. Accessed December 3, 2015.
  9. Fleischer DM, Burks AW. Pitfalls in food allergy diagnosis: serum IgE testing. J Pediatr 2015; 166: 8-10.
  10. Boyce JA, Assa'ad A, Burks AW, et al; NIAID-Sponsored Expert Panel. Guidelines for the diagnosis and management of food allergy in the United States: summary of the NIAID-sponsored expert panel report. J Allergy Clin Immunol 2010; 126:1105–1118.
  11. Stiefel G, Roberts G. How to use serum-specific IgE measurements in diagnosing and monitoring food allergy. Arch Dis Child Educ Pract Ed 2012; 97:29–36.
Article PDF
Author and Disclosure Information

Cheryl K. Lau, PhD
Calgary Laboratory Services, Calgary, Alberta, Canada

Christopher Naugler, MD, CCFP, FCFP, FRCPC
Calgary Laboratory Services, Calgary, Alberta, Canada; Division Head, General Pathology, Department of Pathology and Laboratory Medicine, Department of Family Medicine, Associate Professor, University of Calgary, Alberta, Canada

Address: Christopher Naugler, MD, Calgary Laboratory Services, C410, Diagnostic and Scientific Centre, 9, 3535 Research Road NW, Calgary, AB, Canada T2L 2K8; e-mail: Christopher.naugler@cls.ab.ca

Dr. Naugler has disclosed performing informatics consulting for Abbott Laboratories.

Issue
Cleveland Clinic Journal of Medicine - 83(1)
Publications
Topics
Page Number
21-24
Legacy Keywords
allergy, allergy testing, immunoglobulin E, IgE, blood testing, skin prick testing, Cheryl Lau, Christopher Naugler, smart testing
Sections
Author and Disclosure Information

Cheryl K. Lau, PhD
Calgary Laboratory Services, Calgary, Alberta, Canada

Christopher Naugler, MD, CCFP, FCFP, FRCPC
Calgary Laboratory Services, Calgary, Alberta, Canada; Division Head, General Pathology, Department of Pathology and Laboratory Medicine, Department of Family Medicine, Associate Professor, University of Calgary, Alberta, Canada

Address: Christopher Naugler, MD, Calgary Laboratory Services, C410, Diagnostic and Scientific Centre, 9, 3535 Research Road NW, Calgary, AB, Canada T2L 2K8; e-mail: Christopher.naugler@cls.ab.ca

Dr. Naugler has disclosed performing informatics consulting for Abbott Laboratories.

Author and Disclosure Information

Cheryl K. Lau, PhD
Calgary Laboratory Services, Calgary, Alberta, Canada

Christopher Naugler, MD, CCFP, FCFP, FRCPC
Calgary Laboratory Services, Calgary, Alberta, Canada; Division Head, General Pathology, Department of Pathology and Laboratory Medicine, Department of Family Medicine, Associate Professor, University of Calgary, Alberta, Canada

Address: Christopher Naugler, MD, Calgary Laboratory Services, C410, Diagnostic and Scientific Centre, 9, 3535 Research Road NW, Calgary, AB, Canada T2L 2K8; e-mail: Christopher.naugler@cls.ab.ca

Dr. Naugler has disclosed performing informatics consulting for Abbott Laboratories.

Article PDF
Article PDF
Related Articles

A 25-year-old man is evaluated for angioedema (swelling of lips and tongue) after eating paella at a Spanish restaurant. He has no history of allergies, but he says he had never eaten such a large variety of seafood before, especially shellfish.

He suspects that he is allergic to shellfish and asks the attending physician to order blood tests for seafood allergies, as he heard from a friend that blood tests are superior to other types of tests for allergy. The physician requests a serum immunoglobulin E (IgE) food panel test for this patient.

SERUM ALLERGEN-SPECIFIC IgE TESTING

Many methods of testing for allergy are available, including the skin-prick test, double-blind and single-blind placebo-controlled food challenges, open food challenges, inhalant challenges, drug challenges, and serum IgE tests. In clinical practice, these tests are often used in combination because when used individually, few of them are both highly sensitive and specific (Table 1).1–6

Skin-prick testing is generally the method of choice for the preliminary evaluation of IgE-mediated allergies because it is more sensitive and requires less time to get a result.1 But it is not the preferred test if the patient is at risk of a systemic reaction or has widespread dermatitis, nor is it useful if the patient is taking drugs that suppress the histamine response, such as antihistamines or tricyclic antidepressants.6 Moreover, skin-prick testing is more invasive and time-consuming than serum IgE testing.

Serum IgE testing is an attractive alternative, and it is more convenient because it requires only a single blood draw and poses a lower risk of adverse effects.

NOT A RELIABLE DIAGNOSTIC TOOL

As serum IgE testing has gained popularity, researchers have tried to improve its diagnostic power (ie, maximize its sensitivity and specificity) by determining the best cutoff values for IgE against specific antigens. Unfortunately, these values are difficult to determine because of confounding factors such as the lack of a reference standard, population diversity, patient atopy, and the overwhelming number of allergens that must be examined.

In addition, some researchers have used positive and negative predictive values to evaluate diagnostic cutoffs for serum antigen-specific IgE values. But these are not the most suitable performance measure to evaluate because they depend on disease prevalence and population characteristics.

Despite these efforts, results are still conflicting, and serum antigen-specific IgE testing is not a reliable diagnostic tool.

Figure 1. Sum of sensitivity and specificity of serum antigen-specific IgE tests of different ImmunoCAP allergens. A sum of 170 or greater (dashed line) is considered clinically relevant; tests with IgE cutoffs greater than 0.35 kU/L are noted with an asterisk.

In an effort to gain insight from the available research data, we evaluated the clinical usefulness of 89 antigen-specific IgE tests, using an approach of summing their sensitivity and specificity. Previously, Wians7 proposed that a test is likely to be clinically useful if the sum of its sensitivity and specificity is equal to or greater than 170. Figure 1 shows the 89 tests, grouped into categories, and their summed sensitivities and specificities. The dashed line indicates a cutoff of 170; any bar that touches or crosses that line indicates that the test may be clinically useful, according to Wians.7

Only 7 of the 89 tests (cow, buckwheat, hazelnut, latex, Alternaria alternata, honey bee venom, and Johnson grass) satisfied this criterion. This suggests that a significant number of serum antigen-specific IgE tests perform suboptimally, and we are left with the question of why they are so commonly ordered.

Inappropriate use can lead to false-positive results, a situation in which patients may be subjected to unnecessary food avoidance that can result in nutritional deficiencies and decreased quality of life. It can also lead to false-negative results, when life-threatening diagnoses are missed and further excessive downstream testing is required—all leading; to negative outcomes for both patients and healthcare providers.

CHOOSING WISELY

The Choosing Wisely campaign in the United States has partnered with the American Academy of Allergy, Asthma, and Immunology to advocate against indiscriminate IgE testing in evaluating allergy.8 Allergy diagnosis and evaluation should be based on a combination of clinical history and judicious ordering of specific IgE tests, whether through skin or blood testing. Ordering of serum allergen-specific IgE tests for food allergies should be consistent with a clinical history of potential IgE-mediated food allergy8 and not food intolerance (Table 2).4,5

Some jurisdictions in Canada have followed suit by restricting the number of serum IgE tests each physician is allowed to order per patient, to encourage more responsible ordering and to lower the number of potential false-positive results, which can lead to increased downstream costs as well as unnecessary patient worry and lifestyle modification.

 

 

CLINICAL BOTTOM LINE

Ordering diagnostic tests that have little clinical utility has long-term detrimental effects on both patient safety and healthcare sustainability.

In the case of the 25-year-old evaluated for shellfish allergy, the clinician should first explain that the swelling of the lips and tongue (angioedema) does suggest an IgE-mediated allergic reaction and not a non–IgE-mediated allergic reaction or a food intolerance. Non–IgE-mediated food allergies and food intolerances are marked by symptoms relating mainly to nonimmune aspects of the digestive system, whereas IgE-mediated food allergies affect the immune system and can involve a multitude of organs, including the skin and the respiratory and digestive systems (Table 2).

However, clinicians should avoid indiscriminately ordering food allergen IgE panels and instead should focus on foods likely to be the culprits based on the clinical history.9 Indiscriminate testing can lead to false-positive results and unnecessary food avoidance.

Since the patient developed symptoms of angioedema when he was exposed to his allergen, he may be apprehensive about a skin- prick test and the possibility of being subjected to the same discomfort. Therefore, in this situation, it may be best to perform serum IgE tests, but on a few targeted seafoods rather than the food panel the physician had ordered. A patient can be sensitized to an allergen (possess IgE antibodies) but not experience symptoms when exposed to it (ie, have tolerance).5 Also, false-negative results may occur, so a negative serum allergen-specific IgE test should likewise be interpreted in light of the pretest probability of allergy to a specific antigen.

If the history and the results of testing are not clear and congruent, the patient should be referred to an allergist for diagnosis or for management. The allergist can provide management techniques and periodic assessment as to the progression and resolution of the allergy. Table 2 highlights symptoms that differentiate an IgE-mediated from a non–IgE-mediated food allergy.10,11Table 1 presents clinical indications and suggested diagnostic methods to the five most common allergen groups and the diagnostically invalid tests.1–6

The bottom line is that we must consider the poor performance of serum allergen-specific IgE tests when diagnosing and treating suspected type I allergies and avoid ordering food allergen IgE panels whenever possible.

A 25-year-old man is evaluated for angioedema (swelling of lips and tongue) after eating paella at a Spanish restaurant. He has no history of allergies, but he says he had never eaten such a large variety of seafood before, especially shellfish.

He suspects that he is allergic to shellfish and asks the attending physician to order blood tests for seafood allergies, as he heard from a friend that blood tests are superior to other types of tests for allergy. The physician requests a serum immunoglobulin E (IgE) food panel test for this patient.

SERUM ALLERGEN-SPECIFIC IgE TESTING

Many methods of testing for allergy are available, including the skin-prick test, double-blind and single-blind placebo-controlled food challenges, open food challenges, inhalant challenges, drug challenges, and serum IgE tests. In clinical practice, these tests are often used in combination because when used individually, few of them are both highly sensitive and specific (Table 1).1–6

Skin-prick testing is generally the method of choice for the preliminary evaluation of IgE-mediated allergies because it is more sensitive and requires less time to get a result.1 But it is not the preferred test if the patient is at risk of a systemic reaction or has widespread dermatitis, nor is it useful if the patient is taking drugs that suppress the histamine response, such as antihistamines or tricyclic antidepressants.6 Moreover, skin-prick testing is more invasive and time-consuming than serum IgE testing.

Serum IgE testing is an attractive alternative, and it is more convenient because it requires only a single blood draw and poses a lower risk of adverse effects.

NOT A RELIABLE DIAGNOSTIC TOOL

As serum IgE testing has gained popularity, researchers have tried to improve its diagnostic power (ie, maximize its sensitivity and specificity) by determining the best cutoff values for IgE against specific antigens. Unfortunately, these values are difficult to determine because of confounding factors such as the lack of a reference standard, population diversity, patient atopy, and the overwhelming number of allergens that must be examined.

In addition, some researchers have used positive and negative predictive values to evaluate diagnostic cutoffs for serum antigen-specific IgE values. But these are not the most suitable performance measure to evaluate because they depend on disease prevalence and population characteristics.

Despite these efforts, results are still conflicting, and serum antigen-specific IgE testing is not a reliable diagnostic tool.

Figure 1. Sum of sensitivity and specificity of serum antigen-specific IgE tests of different ImmunoCAP allergens. A sum of 170 or greater (dashed line) is considered clinically relevant; tests with IgE cutoffs greater than 0.35 kU/L are noted with an asterisk.

In an effort to gain insight from the available research data, we evaluated the clinical usefulness of 89 antigen-specific IgE tests, using an approach of summing their sensitivity and specificity. Previously, Wians7 proposed that a test is likely to be clinically useful if the sum of its sensitivity and specificity is equal to or greater than 170. Figure 1 shows the 89 tests, grouped into categories, and their summed sensitivities and specificities. The dashed line indicates a cutoff of 170; any bar that touches or crosses that line indicates that the test may be clinically useful, according to Wians.7

Only 7 of the 89 tests (cow, buckwheat, hazelnut, latex, Alternaria alternata, honey bee venom, and Johnson grass) satisfied this criterion. This suggests that a significant number of serum antigen-specific IgE tests perform suboptimally, and we are left with the question of why they are so commonly ordered.

Inappropriate use can lead to false-positive results, a situation in which patients may be subjected to unnecessary food avoidance that can result in nutritional deficiencies and decreased quality of life. It can also lead to false-negative results, when life-threatening diagnoses are missed and further excessive downstream testing is required—all leading; to negative outcomes for both patients and healthcare providers.

CHOOSING WISELY

The Choosing Wisely campaign in the United States has partnered with the American Academy of Allergy, Asthma, and Immunology to advocate against indiscriminate IgE testing in evaluating allergy.8 Allergy diagnosis and evaluation should be based on a combination of clinical history and judicious ordering of specific IgE tests, whether through skin or blood testing. Ordering of serum allergen-specific IgE tests for food allergies should be consistent with a clinical history of potential IgE-mediated food allergy8 and not food intolerance (Table 2).4,5

Some jurisdictions in Canada have followed suit by restricting the number of serum IgE tests each physician is allowed to order per patient, to encourage more responsible ordering and to lower the number of potential false-positive results, which can lead to increased downstream costs as well as unnecessary patient worry and lifestyle modification.

 

 

CLINICAL BOTTOM LINE

Ordering diagnostic tests that have little clinical utility has long-term detrimental effects on both patient safety and healthcare sustainability.

In the case of the 25-year-old evaluated for shellfish allergy, the clinician should first explain that the swelling of the lips and tongue (angioedema) does suggest an IgE-mediated allergic reaction and not a non–IgE-mediated allergic reaction or a food intolerance. Non–IgE-mediated food allergies and food intolerances are marked by symptoms relating mainly to nonimmune aspects of the digestive system, whereas IgE-mediated food allergies affect the immune system and can involve a multitude of organs, including the skin and the respiratory and digestive systems (Table 2).

However, clinicians should avoid indiscriminately ordering food allergen IgE panels and instead should focus on foods likely to be the culprits based on the clinical history.9 Indiscriminate testing can lead to false-positive results and unnecessary food avoidance.

Since the patient developed symptoms of angioedema when he was exposed to his allergen, he may be apprehensive about a skin- prick test and the possibility of being subjected to the same discomfort. Therefore, in this situation, it may be best to perform serum IgE tests, but on a few targeted seafoods rather than the food panel the physician had ordered. A patient can be sensitized to an allergen (possess IgE antibodies) but not experience symptoms when exposed to it (ie, have tolerance).5 Also, false-negative results may occur, so a negative serum allergen-specific IgE test should likewise be interpreted in light of the pretest probability of allergy to a specific antigen.

If the history and the results of testing are not clear and congruent, the patient should be referred to an allergist for diagnosis or for management. The allergist can provide management techniques and periodic assessment as to the progression and resolution of the allergy. Table 2 highlights symptoms that differentiate an IgE-mediated from a non–IgE-mediated food allergy.10,11Table 1 presents clinical indications and suggested diagnostic methods to the five most common allergen groups and the diagnostically invalid tests.1–6

The bottom line is that we must consider the poor performance of serum allergen-specific IgE tests when diagnosing and treating suspected type I allergies and avoid ordering food allergen IgE panels whenever possible.

References
  1. Bernstein IL, Li JT, Bernstein DI, et al; American Academy of Allergy, Asthma and Immunology; American College of Allergy, Asthma and Immunology. Allergy diagnostic testing: an updated practice parameter. Ann Allergy Asthma Immunol 2008; 100(suppl 3):S1–S148.
  2. Bird JA, Crain M, Varshney P. Food allergen panel testing often results in misdiagnosis of food allergy. J Pediatr 2015; 166:97–100.
  3. Kattan JD, Sicherer SH. Optimizing the diagnosis of food allergy. Immunol Allergy Clin North Am 2015; 35:61–76.
  4. Sampson HA, Aceves S, Bock SA, et al. Food allergy: a practice parameter update-2014. J Allergy Clin Immunol 2014; 134:1016–1025.e43.
  5. Sicherer SH, Sampson HA. Food allergy: epidemiology, pathogenesis, diagnosis, and treatment. J Allergy Clin Immunol 2014; 133:291–308.
  6. Siles RI, Hsieh FH. Allergy blood testing: a practical guide for clinicians. Cleve Clin J Med 2011; 78:585–592.
  7. Wians FH Jr. Clinical laboratory tests: which, why, and what do the results mean? Lab Medicine 2009; 40:105–113.
  8. Choosing Wisely. American Academy of Allergy, Asthma & Immunology. Ten Things Physicians and Patients Should Question. www.choosingwisely.org/doctor-patient-lists/american-academy-of-allergy-asthma-immunology/. Accessed December 3, 2015.
  9. Fleischer DM, Burks AW. Pitfalls in food allergy diagnosis: serum IgE testing. J Pediatr 2015; 166: 8-10.
  10. Boyce JA, Assa'ad A, Burks AW, et al; NIAID-Sponsored Expert Panel. Guidelines for the diagnosis and management of food allergy in the United States: summary of the NIAID-sponsored expert panel report. J Allergy Clin Immunol 2010; 126:1105–1118.
  11. Stiefel G, Roberts G. How to use serum-specific IgE measurements in diagnosing and monitoring food allergy. Arch Dis Child Educ Pract Ed 2012; 97:29–36.
References
  1. Bernstein IL, Li JT, Bernstein DI, et al; American Academy of Allergy, Asthma and Immunology; American College of Allergy, Asthma and Immunology. Allergy diagnostic testing: an updated practice parameter. Ann Allergy Asthma Immunol 2008; 100(suppl 3):S1–S148.
  2. Bird JA, Crain M, Varshney P. Food allergen panel testing often results in misdiagnosis of food allergy. J Pediatr 2015; 166:97–100.
  3. Kattan JD, Sicherer SH. Optimizing the diagnosis of food allergy. Immunol Allergy Clin North Am 2015; 35:61–76.
  4. Sampson HA, Aceves S, Bock SA, et al. Food allergy: a practice parameter update-2014. J Allergy Clin Immunol 2014; 134:1016–1025.e43.
  5. Sicherer SH, Sampson HA. Food allergy: epidemiology, pathogenesis, diagnosis, and treatment. J Allergy Clin Immunol 2014; 133:291–308.
  6. Siles RI, Hsieh FH. Allergy blood testing: a practical guide for clinicians. Cleve Clin J Med 2011; 78:585–592.
  7. Wians FH Jr. Clinical laboratory tests: which, why, and what do the results mean? Lab Medicine 2009; 40:105–113.
  8. Choosing Wisely. American Academy of Allergy, Asthma & Immunology. Ten Things Physicians and Patients Should Question. www.choosingwisely.org/doctor-patient-lists/american-academy-of-allergy-asthma-immunology/. Accessed December 3, 2015.
  9. Fleischer DM, Burks AW. Pitfalls in food allergy diagnosis: serum IgE testing. J Pediatr 2015; 166: 8-10.
  10. Boyce JA, Assa'ad A, Burks AW, et al; NIAID-Sponsored Expert Panel. Guidelines for the diagnosis and management of food allergy in the United States: summary of the NIAID-sponsored expert panel report. J Allergy Clin Immunol 2010; 126:1105–1118.
  11. Stiefel G, Roberts G. How to use serum-specific IgE measurements in diagnosing and monitoring food allergy. Arch Dis Child Educ Pract Ed 2012; 97:29–36.
Issue
Cleveland Clinic Journal of Medicine - 83(1)
Issue
Cleveland Clinic Journal of Medicine - 83(1)
Page Number
21-24
Page Number
21-24
Publications
Publications
Topics
Article Type
Display Headline
Serum allergen-specific IgE testing: How much is too much?
Display Headline
Serum allergen-specific IgE testing: How much is too much?
Legacy Keywords
allergy, allergy testing, immunoglobulin E, IgE, blood testing, skin prick testing, Cheryl Lau, Christopher Naugler, smart testing
Legacy Keywords
allergy, allergy testing, immunoglobulin E, IgE, blood testing, skin prick testing, Cheryl Lau, Christopher Naugler, smart testing
Sections
Disallow All Ads
Alternative CME
Article PDF Media

Do healthy patients need routine laboratory testing before elective noncardiac surgery?

Article Type
Changed
Tue, 09/12/2017 - 12:36
Display Headline
Do healthy patients need routine laboratory testing before elective noncardiac surgery?

A 63-year-old physician is referred for preoperative evaluation before arthroscopic repair of a torn medial meniscus. Her exercise tolerance was excellent before the knee injury, including running without cardiopulmonary symptoms. She is otherwise healthy except for hypertension that is well controlled on amlodipine. She has no known history of liver or kidney disease, bleeding disorder, recent illness, or complications with anesthesia. She inquires as to whether “routine blood testing” is needed before the procedure.

See related editorial

What laboratory studies, if any, should be ordered?

UNLIKELY TO BE OF BENEFIT

Preoperative laboratory testing is not necessary in this otherwise healthy, asymptomatic patient. In the absence of clinical indications, routine testing before elective, low-risk procedures often increases both the cost of care and the potential anxiety caused by abnormal results that provide no substantial benefit to the patient or the clinician.

Preoperative diagnostic tests should be ordered only to identify and optimize disorders that alter the likelihood of perioperative and postoperative adverse outcomes and to establish a baseline assessment. Yet clinicians often perceive that laboratory testing is required by their organization or by other providers.

A comprehensive history and physical examination are the cornerstones of the effective preoperative evaluation. Preferably, the history and examination should guide further testing rather than ordering a battery of standard tests for all patients. However, selective preoperative laboratory testing may be useful in certain situations, such as in patients undergoing high-risk procedures and those with known underlying conditions or factors that may affect operative management (Table 1).

Unfortunately, high-quality evidence for this selective approach is lacking. According to one observational study,1 when laboratory testing is appropriate, it is reasonable to use test results already obtained and normal within the preceding 4 months unless the patient has had an interim change in health status.

Definitions of risk stratification (eg, urgency of surgical procedure, graded risk according to type of operation) and tools such as the Revised Cardiac Risk Index can be found in the 2014 American College of Cardiology/American Heart Association guidelines2 and may be useful to distinguish healthy patients from those with significant comorbidities, as well as to distinguish low-risk, elective procedures from those that impart higher risk.

Professional societies and guidelines in many countries have criticized the habitual practice of extensive, nonselective laboratory testing.3–6 Yet despite lack of evidence of benefit, routine preoperative testing is still often done. At an estimated cost of more than $18 billion in the United States annually,7 preoperative testing deserves attention, especially in this time of ballooning healthcare costs and increased focus on effective and efficient care.

EVIDENCE AND GUIDELINES

Numerous studies have established that routine laboratory testing rarely changes the preoperative management of the patient or improves surgical outcomes. Narr et al8 found that 160 (4%) of 3,782 patients who underwent ambulatory surgery had abnormal test results, and only 10 required treatment. In this study, there was no association between abnormal test results and perioperative management or postoperative adverse events.

In a systematic review, Smetana and Macpherson9 noted that the incidence of laboratory test abnormalities that led to a change in management ranged from 0.1% to 2.6%. Notably, clinicians ignore 30% to 60% of abnormal preoperative laboratory results, a practice that may create additional medicolegal risk.7

Most guidelines on preoperative testing are based on expert opinion, case series, or consensus

Little evidence exists that helps in the development of guidelines for preoperative laboratory testing. Most guidelines are based on expert opinion, case series, and consensus. As an example of the heterogeneity this creates, the American Society of Anesthesiologists, the Ontario Preoperative Testing Group, and the Canadian Anesthesiologists’ Society provide different recommended indications for preoperative laboratory testing in patients with “advanced age” but do not define a clear minimum age for this cohort.10

However, one area that does have substantial data is cataract surgery. Patients in their usual state of health who are to undergo this procedure do not require preoperative testing, a claim supported by high-quality evidence including a 2012 Cochrane systematic review.11

Munro et al5 performed a systematic review of the evidence behind preoperative laboratory testing, concluding that the power of preoperative tests to predict adverse postoperative outcomes in asymptomatic patients is either weak or nonexistent. The National Institute for Health and Clinical Excellence guidelines of 2003,6 the Practice Advisory for Preanesthesia Evaluation of the American Society of Anesthesiologists of 2012,12 the Institute for Clinical Systems Improvement guideline of 2012,13 and a systematic review conducted by Johansson et al14 found no evidence from high-quality studies to support the claim that routine preoperative testing is beneficial in healthy adults undergoing noncardiac surgery, but that certain patient populations may benefit from selective testing.

A randomized controlled trial evaluated the elimination of preoperative testing in patients undergoing low-risk ambulatory surgery and found no difference in perioperative adverse events in the control and intervention arms.15 Similar studies achieved the same results.

The Choosing Wisely campaign

The American Board of Internal Medicine Foundation has partnered with medical specialty societies to create lists of common practice patterns that should be questioned and possibly discontinued. These lists are collectively called the Choosing Wisely campaign (www.choosingwisely.org). Avoiding routine preoperative laboratory testing in patients undergoing low-risk surgery without clinical indications can be found in the lists for the American Society of Anesthesiologists, the American Society for Clinical Pathology, and the Society of General Internal Medicine.

THE POSSIBLE HARMS OF TESTING

The prevalence of unrecognized disease that influences the risk of surgery in healthy patients is low, and thus the predictive value of abnormal test values in these patients is low. This leads to substantial false-positivity, which is of uncertain clinical significance and which may in turn cause a cascade of further testing. Not surprisingly, the probability of an abnormal test result increases dramatically with the number of tests ordered, a fact that magnifies the problem of false-positive results.

The costs and harms associated with testing are both direct and indirect. Direct effects include increased healthcare costs of further testing or potentially unnecessary treatment as well as risk associated with additional testing, though these are not common, as there is a low (< 3%) incidence of a change in preoperative management based on an abnormal test result. Likewise, normal results do not appear to substantially reduce the likelihood of postoperative complications.9

Indirect effects, which are particularly challenging to measure, may include time lost from employment to pursue further evaluation and anxiety surrounding abnormal results.

THE CLINICAL BOTTOM LINE

Based on over 2 decades of data, our 63-year-old patient should not undergo “routine” preoperative laboratory testing before her upcoming elective, low-risk, noncardiac procedure. Her hypertension is well controlled, and she is taking no medications that may lead to clinically significant metabolic derangements or significant changes in surgical outcome. There are no convincing clinical indications for further laboratory investigation. Further, the results are unlikely to affect the preoperative management and rate of adverse events; the direct and indirect costs may be substantial; and there is a small but tangible risk of harm.

Given the myriad factors that influence unnecessary preoperative testing, a focus on systems-level solutions is paramount. Key steps may include creation and adoption of clear and consistent guidelines, development of clinical care pathways, physician education and modification of practice, interdisciplinary communication and information sharing, economic analysis, and outcomes assessment.

References
  1. Macpherson DS, Snow R, Lofgren RP. Preoperative screening: value of previous tests. Ann Intern Med 1990; 113:969–973.
  2. Fleisher LA, Fleischmann KE, Auerbach AD, et al. 2014 ACC/AHA guideline on perioperative cardiovascular evaluation and management of patients undergoing noncardiac surgery. Circulation 2014; 130:e278–e333.
  3. Schein OD, Katz J, Bass EB, et al. The value of routine preoperative medical testing before cataract surgery. Study of medical testing for cataract surgery. N Engl J Med 2000; 342:168–175.
  4. The Swedish Council on Technology Assessment in Health Care (SBU). Preoperative routines. Stockholm, 1989.
  5. Munro J, Booth A, Nicholl J. Routine preoperative testing: a systematic review of the evidence. Health Technol Assess 1997; 1:1–62.
  6. National Institute for Health and Clinical Excellence (NICE). Preoperative tests: The use of routine preoperative tests for elective surgery. London: National Collaborating Centre for Acute Care, 2003.
  7. Roizen MF. More preoperative assessment by physicians and less by laboratory tests. N Engl J Med 2000; 342:204–205.
  8. Narr BJ, Hansen TR, Warner MA. Preoperative laboratory screening in healthy Mayo patients: cost-effective elimination of tests and unchanged outcomes. Mayo Clin Proc 1991; 66:155–159.
  9. Smetana GW, Macpherson DS. The case against routine preoperative laboratory testing. Med Clin North Am 2003; 87:7–40.
  10. Benarroch-Gampel J, Sheffield KM, Duncan CB, et al. Preoperative laboratory testing in patients undergoing elective, low-risk ambulatory surgery. Ann Surg 2012; 256:518–528.
  11. Keay L, Lindsley K, Tielsch J, Katz J, Schein O. Routine preoperative medical testing for cataract surgery. Cochrane Database Syst Rev 2012; 3:CD007293.
  12. Committee on Standards and Practice Parameters; Apfelbaum JL, Connis RT, Nickinovich DG, et al. Practice advisory for preanesthesia evaluation: an updated report by the American Society of Anesthesiologists Task Force on Preanesthesia Evaluation. Anesthesiology 2012; 116:522–538.
  13. Institute for Clinical Systems Improvement (ICSI). Health care guideline: preoperative evaluation. 10th ed. Bloomington, MN: Institute for Clinical Systems Improvement; 2012.
  14. Johansson T, Fritsch G, Flamm M, et al. Effectiveness of non-cardiac preoperative testing in non-cardiac elective surgery: a systematic review. Br J Anaesth 2013; 110:926–939.
  15. Chung F, Yuan H, Yin L, Vairavanathan S, Wong DT. Elimination of preoperative testing in ambulatory surgery. Anesth Analg 2009; 108:467–475.
Article PDF
Author and Disclosure Information

Nathan Houchens, MD, FACP
Department of Hospital Medicine, Medicine Institute, Cleveland Clinic; Assistant Professor, Cleveland Clinic Lerner College of Medicine of Case Western Reserve University, Cleveland, OH

Address: Nathan Houchens, MD, FACP, Department of Hospital Medicine, M2 Annex, Cleveland Clinic, 9500 Euclid Avenue, Cleveland, OH 44195; e-mail: houchen@ccf.org

Issue
Cleveland Clinic Journal of Medicine - 82(10)
Publications
Topics
Page Number
664-666
Legacy Keywords
preoperative testing, Smart testing, guidelines, evidence-based medicine, defensive medicine, Nathan Houchens
Sections
Author and Disclosure Information

Nathan Houchens, MD, FACP
Department of Hospital Medicine, Medicine Institute, Cleveland Clinic; Assistant Professor, Cleveland Clinic Lerner College of Medicine of Case Western Reserve University, Cleveland, OH

Address: Nathan Houchens, MD, FACP, Department of Hospital Medicine, M2 Annex, Cleveland Clinic, 9500 Euclid Avenue, Cleveland, OH 44195; e-mail: houchen@ccf.org

Author and Disclosure Information

Nathan Houchens, MD, FACP
Department of Hospital Medicine, Medicine Institute, Cleveland Clinic; Assistant Professor, Cleveland Clinic Lerner College of Medicine of Case Western Reserve University, Cleveland, OH

Address: Nathan Houchens, MD, FACP, Department of Hospital Medicine, M2 Annex, Cleveland Clinic, 9500 Euclid Avenue, Cleveland, OH 44195; e-mail: houchen@ccf.org

Article PDF
Article PDF
Related Articles

A 63-year-old physician is referred for preoperative evaluation before arthroscopic repair of a torn medial meniscus. Her exercise tolerance was excellent before the knee injury, including running without cardiopulmonary symptoms. She is otherwise healthy except for hypertension that is well controlled on amlodipine. She has no known history of liver or kidney disease, bleeding disorder, recent illness, or complications with anesthesia. She inquires as to whether “routine blood testing” is needed before the procedure.

See related editorial

What laboratory studies, if any, should be ordered?

UNLIKELY TO BE OF BENEFIT

Preoperative laboratory testing is not necessary in this otherwise healthy, asymptomatic patient. In the absence of clinical indications, routine testing before elective, low-risk procedures often increases both the cost of care and the potential anxiety caused by abnormal results that provide no substantial benefit to the patient or the clinician.

Preoperative diagnostic tests should be ordered only to identify and optimize disorders that alter the likelihood of perioperative and postoperative adverse outcomes and to establish a baseline assessment. Yet clinicians often perceive that laboratory testing is required by their organization or by other providers.

A comprehensive history and physical examination are the cornerstones of the effective preoperative evaluation. Preferably, the history and examination should guide further testing rather than ordering a battery of standard tests for all patients. However, selective preoperative laboratory testing may be useful in certain situations, such as in patients undergoing high-risk procedures and those with known underlying conditions or factors that may affect operative management (Table 1).

Unfortunately, high-quality evidence for this selective approach is lacking. According to one observational study,1 when laboratory testing is appropriate, it is reasonable to use test results already obtained and normal within the preceding 4 months unless the patient has had an interim change in health status.

Definitions of risk stratification (eg, urgency of surgical procedure, graded risk according to type of operation) and tools such as the Revised Cardiac Risk Index can be found in the 2014 American College of Cardiology/American Heart Association guidelines2 and may be useful to distinguish healthy patients from those with significant comorbidities, as well as to distinguish low-risk, elective procedures from those that impart higher risk.

Professional societies and guidelines in many countries have criticized the habitual practice of extensive, nonselective laboratory testing.3–6 Yet despite lack of evidence of benefit, routine preoperative testing is still often done. At an estimated cost of more than $18 billion in the United States annually,7 preoperative testing deserves attention, especially in this time of ballooning healthcare costs and increased focus on effective and efficient care.

EVIDENCE AND GUIDELINES

Numerous studies have established that routine laboratory testing rarely changes the preoperative management of the patient or improves surgical outcomes. Narr et al8 found that 160 (4%) of 3,782 patients who underwent ambulatory surgery had abnormal test results, and only 10 required treatment. In this study, there was no association between abnormal test results and perioperative management or postoperative adverse events.

In a systematic review, Smetana and Macpherson9 noted that the incidence of laboratory test abnormalities that led to a change in management ranged from 0.1% to 2.6%. Notably, clinicians ignore 30% to 60% of abnormal preoperative laboratory results, a practice that may create additional medicolegal risk.7

Most guidelines on preoperative testing are based on expert opinion, case series, or consensus

Little evidence exists that helps in the development of guidelines for preoperative laboratory testing. Most guidelines are based on expert opinion, case series, and consensus. As an example of the heterogeneity this creates, the American Society of Anesthesiologists, the Ontario Preoperative Testing Group, and the Canadian Anesthesiologists’ Society provide different recommended indications for preoperative laboratory testing in patients with “advanced age” but do not define a clear minimum age for this cohort.10

However, one area that does have substantial data is cataract surgery. Patients in their usual state of health who are to undergo this procedure do not require preoperative testing, a claim supported by high-quality evidence including a 2012 Cochrane systematic review.11

Munro et al5 performed a systematic review of the evidence behind preoperative laboratory testing, concluding that the power of preoperative tests to predict adverse postoperative outcomes in asymptomatic patients is either weak or nonexistent. The National Institute for Health and Clinical Excellence guidelines of 2003,6 the Practice Advisory for Preanesthesia Evaluation of the American Society of Anesthesiologists of 2012,12 the Institute for Clinical Systems Improvement guideline of 2012,13 and a systematic review conducted by Johansson et al14 found no evidence from high-quality studies to support the claim that routine preoperative testing is beneficial in healthy adults undergoing noncardiac surgery, but that certain patient populations may benefit from selective testing.

A randomized controlled trial evaluated the elimination of preoperative testing in patients undergoing low-risk ambulatory surgery and found no difference in perioperative adverse events in the control and intervention arms.15 Similar studies achieved the same results.

The Choosing Wisely campaign

The American Board of Internal Medicine Foundation has partnered with medical specialty societies to create lists of common practice patterns that should be questioned and possibly discontinued. These lists are collectively called the Choosing Wisely campaign (www.choosingwisely.org). Avoiding routine preoperative laboratory testing in patients undergoing low-risk surgery without clinical indications can be found in the lists for the American Society of Anesthesiologists, the American Society for Clinical Pathology, and the Society of General Internal Medicine.

THE POSSIBLE HARMS OF TESTING

The prevalence of unrecognized disease that influences the risk of surgery in healthy patients is low, and thus the predictive value of abnormal test values in these patients is low. This leads to substantial false-positivity, which is of uncertain clinical significance and which may in turn cause a cascade of further testing. Not surprisingly, the probability of an abnormal test result increases dramatically with the number of tests ordered, a fact that magnifies the problem of false-positive results.

The costs and harms associated with testing are both direct and indirect. Direct effects include increased healthcare costs of further testing or potentially unnecessary treatment as well as risk associated with additional testing, though these are not common, as there is a low (< 3%) incidence of a change in preoperative management based on an abnormal test result. Likewise, normal results do not appear to substantially reduce the likelihood of postoperative complications.9

Indirect effects, which are particularly challenging to measure, may include time lost from employment to pursue further evaluation and anxiety surrounding abnormal results.

THE CLINICAL BOTTOM LINE

Based on over 2 decades of data, our 63-year-old patient should not undergo “routine” preoperative laboratory testing before her upcoming elective, low-risk, noncardiac procedure. Her hypertension is well controlled, and she is taking no medications that may lead to clinically significant metabolic derangements or significant changes in surgical outcome. There are no convincing clinical indications for further laboratory investigation. Further, the results are unlikely to affect the preoperative management and rate of adverse events; the direct and indirect costs may be substantial; and there is a small but tangible risk of harm.

Given the myriad factors that influence unnecessary preoperative testing, a focus on systems-level solutions is paramount. Key steps may include creation and adoption of clear and consistent guidelines, development of clinical care pathways, physician education and modification of practice, interdisciplinary communication and information sharing, economic analysis, and outcomes assessment.

A 63-year-old physician is referred for preoperative evaluation before arthroscopic repair of a torn medial meniscus. Her exercise tolerance was excellent before the knee injury, including running without cardiopulmonary symptoms. She is otherwise healthy except for hypertension that is well controlled on amlodipine. She has no known history of liver or kidney disease, bleeding disorder, recent illness, or complications with anesthesia. She inquires as to whether “routine blood testing” is needed before the procedure.

See related editorial

What laboratory studies, if any, should be ordered?

UNLIKELY TO BE OF BENEFIT

Preoperative laboratory testing is not necessary in this otherwise healthy, asymptomatic patient. In the absence of clinical indications, routine testing before elective, low-risk procedures often increases both the cost of care and the potential anxiety caused by abnormal results that provide no substantial benefit to the patient or the clinician.

Preoperative diagnostic tests should be ordered only to identify and optimize disorders that alter the likelihood of perioperative and postoperative adverse outcomes and to establish a baseline assessment. Yet clinicians often perceive that laboratory testing is required by their organization or by other providers.

A comprehensive history and physical examination are the cornerstones of the effective preoperative evaluation. Preferably, the history and examination should guide further testing rather than ordering a battery of standard tests for all patients. However, selective preoperative laboratory testing may be useful in certain situations, such as in patients undergoing high-risk procedures and those with known underlying conditions or factors that may affect operative management (Table 1).

Unfortunately, high-quality evidence for this selective approach is lacking. According to one observational study,1 when laboratory testing is appropriate, it is reasonable to use test results already obtained and normal within the preceding 4 months unless the patient has had an interim change in health status.

Definitions of risk stratification (eg, urgency of surgical procedure, graded risk according to type of operation) and tools such as the Revised Cardiac Risk Index can be found in the 2014 American College of Cardiology/American Heart Association guidelines2 and may be useful to distinguish healthy patients from those with significant comorbidities, as well as to distinguish low-risk, elective procedures from those that impart higher risk.

Professional societies and guidelines in many countries have criticized the habitual practice of extensive, nonselective laboratory testing.3–6 Yet despite lack of evidence of benefit, routine preoperative testing is still often done. At an estimated cost of more than $18 billion in the United States annually,7 preoperative testing deserves attention, especially in this time of ballooning healthcare costs and increased focus on effective and efficient care.

EVIDENCE AND GUIDELINES

Numerous studies have established that routine laboratory testing rarely changes the preoperative management of the patient or improves surgical outcomes. Narr et al8 found that 160 (4%) of 3,782 patients who underwent ambulatory surgery had abnormal test results, and only 10 required treatment. In this study, there was no association between abnormal test results and perioperative management or postoperative adverse events.

In a systematic review, Smetana and Macpherson9 noted that the incidence of laboratory test abnormalities that led to a change in management ranged from 0.1% to 2.6%. Notably, clinicians ignore 30% to 60% of abnormal preoperative laboratory results, a practice that may create additional medicolegal risk.7

Most guidelines on preoperative testing are based on expert opinion, case series, or consensus

Little evidence exists that helps in the development of guidelines for preoperative laboratory testing. Most guidelines are based on expert opinion, case series, and consensus. As an example of the heterogeneity this creates, the American Society of Anesthesiologists, the Ontario Preoperative Testing Group, and the Canadian Anesthesiologists’ Society provide different recommended indications for preoperative laboratory testing in patients with “advanced age” but do not define a clear minimum age for this cohort.10

However, one area that does have substantial data is cataract surgery. Patients in their usual state of health who are to undergo this procedure do not require preoperative testing, a claim supported by high-quality evidence including a 2012 Cochrane systematic review.11

Munro et al5 performed a systematic review of the evidence behind preoperative laboratory testing, concluding that the power of preoperative tests to predict adverse postoperative outcomes in asymptomatic patients is either weak or nonexistent. The National Institute for Health and Clinical Excellence guidelines of 2003,6 the Practice Advisory for Preanesthesia Evaluation of the American Society of Anesthesiologists of 2012,12 the Institute for Clinical Systems Improvement guideline of 2012,13 and a systematic review conducted by Johansson et al14 found no evidence from high-quality studies to support the claim that routine preoperative testing is beneficial in healthy adults undergoing noncardiac surgery, but that certain patient populations may benefit from selective testing.

A randomized controlled trial evaluated the elimination of preoperative testing in patients undergoing low-risk ambulatory surgery and found no difference in perioperative adverse events in the control and intervention arms.15 Similar studies achieved the same results.

The Choosing Wisely campaign

The American Board of Internal Medicine Foundation has partnered with medical specialty societies to create lists of common practice patterns that should be questioned and possibly discontinued. These lists are collectively called the Choosing Wisely campaign (www.choosingwisely.org). Avoiding routine preoperative laboratory testing in patients undergoing low-risk surgery without clinical indications can be found in the lists for the American Society of Anesthesiologists, the American Society for Clinical Pathology, and the Society of General Internal Medicine.

THE POSSIBLE HARMS OF TESTING

The prevalence of unrecognized disease that influences the risk of surgery in healthy patients is low, and thus the predictive value of abnormal test values in these patients is low. This leads to substantial false-positivity, which is of uncertain clinical significance and which may in turn cause a cascade of further testing. Not surprisingly, the probability of an abnormal test result increases dramatically with the number of tests ordered, a fact that magnifies the problem of false-positive results.

The costs and harms associated with testing are both direct and indirect. Direct effects include increased healthcare costs of further testing or potentially unnecessary treatment as well as risk associated with additional testing, though these are not common, as there is a low (< 3%) incidence of a change in preoperative management based on an abnormal test result. Likewise, normal results do not appear to substantially reduce the likelihood of postoperative complications.9

Indirect effects, which are particularly challenging to measure, may include time lost from employment to pursue further evaluation and anxiety surrounding abnormal results.

THE CLINICAL BOTTOM LINE

Based on over 2 decades of data, our 63-year-old patient should not undergo “routine” preoperative laboratory testing before her upcoming elective, low-risk, noncardiac procedure. Her hypertension is well controlled, and she is taking no medications that may lead to clinically significant metabolic derangements or significant changes in surgical outcome. There are no convincing clinical indications for further laboratory investigation. Further, the results are unlikely to affect the preoperative management and rate of adverse events; the direct and indirect costs may be substantial; and there is a small but tangible risk of harm.

Given the myriad factors that influence unnecessary preoperative testing, a focus on systems-level solutions is paramount. Key steps may include creation and adoption of clear and consistent guidelines, development of clinical care pathways, physician education and modification of practice, interdisciplinary communication and information sharing, economic analysis, and outcomes assessment.

References
  1. Macpherson DS, Snow R, Lofgren RP. Preoperative screening: value of previous tests. Ann Intern Med 1990; 113:969–973.
  2. Fleisher LA, Fleischmann KE, Auerbach AD, et al. 2014 ACC/AHA guideline on perioperative cardiovascular evaluation and management of patients undergoing noncardiac surgery. Circulation 2014; 130:e278–e333.
  3. Schein OD, Katz J, Bass EB, et al. The value of routine preoperative medical testing before cataract surgery. Study of medical testing for cataract surgery. N Engl J Med 2000; 342:168–175.
  4. The Swedish Council on Technology Assessment in Health Care (SBU). Preoperative routines. Stockholm, 1989.
  5. Munro J, Booth A, Nicholl J. Routine preoperative testing: a systematic review of the evidence. Health Technol Assess 1997; 1:1–62.
  6. National Institute for Health and Clinical Excellence (NICE). Preoperative tests: The use of routine preoperative tests for elective surgery. London: National Collaborating Centre for Acute Care, 2003.
  7. Roizen MF. More preoperative assessment by physicians and less by laboratory tests. N Engl J Med 2000; 342:204–205.
  8. Narr BJ, Hansen TR, Warner MA. Preoperative laboratory screening in healthy Mayo patients: cost-effective elimination of tests and unchanged outcomes. Mayo Clin Proc 1991; 66:155–159.
  9. Smetana GW, Macpherson DS. The case against routine preoperative laboratory testing. Med Clin North Am 2003; 87:7–40.
  10. Benarroch-Gampel J, Sheffield KM, Duncan CB, et al. Preoperative laboratory testing in patients undergoing elective, low-risk ambulatory surgery. Ann Surg 2012; 256:518–528.
  11. Keay L, Lindsley K, Tielsch J, Katz J, Schein O. Routine preoperative medical testing for cataract surgery. Cochrane Database Syst Rev 2012; 3:CD007293.
  12. Committee on Standards and Practice Parameters; Apfelbaum JL, Connis RT, Nickinovich DG, et al. Practice advisory for preanesthesia evaluation: an updated report by the American Society of Anesthesiologists Task Force on Preanesthesia Evaluation. Anesthesiology 2012; 116:522–538.
  13. Institute for Clinical Systems Improvement (ICSI). Health care guideline: preoperative evaluation. 10th ed. Bloomington, MN: Institute for Clinical Systems Improvement; 2012.
  14. Johansson T, Fritsch G, Flamm M, et al. Effectiveness of non-cardiac preoperative testing in non-cardiac elective surgery: a systematic review. Br J Anaesth 2013; 110:926–939.
  15. Chung F, Yuan H, Yin L, Vairavanathan S, Wong DT. Elimination of preoperative testing in ambulatory surgery. Anesth Analg 2009; 108:467–475.
References
  1. Macpherson DS, Snow R, Lofgren RP. Preoperative screening: value of previous tests. Ann Intern Med 1990; 113:969–973.
  2. Fleisher LA, Fleischmann KE, Auerbach AD, et al. 2014 ACC/AHA guideline on perioperative cardiovascular evaluation and management of patients undergoing noncardiac surgery. Circulation 2014; 130:e278–e333.
  3. Schein OD, Katz J, Bass EB, et al. The value of routine preoperative medical testing before cataract surgery. Study of medical testing for cataract surgery. N Engl J Med 2000; 342:168–175.
  4. The Swedish Council on Technology Assessment in Health Care (SBU). Preoperative routines. Stockholm, 1989.
  5. Munro J, Booth A, Nicholl J. Routine preoperative testing: a systematic review of the evidence. Health Technol Assess 1997; 1:1–62.
  6. National Institute for Health and Clinical Excellence (NICE). Preoperative tests: The use of routine preoperative tests for elective surgery. London: National Collaborating Centre for Acute Care, 2003.
  7. Roizen MF. More preoperative assessment by physicians and less by laboratory tests. N Engl J Med 2000; 342:204–205.
  8. Narr BJ, Hansen TR, Warner MA. Preoperative laboratory screening in healthy Mayo patients: cost-effective elimination of tests and unchanged outcomes. Mayo Clin Proc 1991; 66:155–159.
  9. Smetana GW, Macpherson DS. The case against routine preoperative laboratory testing. Med Clin North Am 2003; 87:7–40.
  10. Benarroch-Gampel J, Sheffield KM, Duncan CB, et al. Preoperative laboratory testing in patients undergoing elective, low-risk ambulatory surgery. Ann Surg 2012; 256:518–528.
  11. Keay L, Lindsley K, Tielsch J, Katz J, Schein O. Routine preoperative medical testing for cataract surgery. Cochrane Database Syst Rev 2012; 3:CD007293.
  12. Committee on Standards and Practice Parameters; Apfelbaum JL, Connis RT, Nickinovich DG, et al. Practice advisory for preanesthesia evaluation: an updated report by the American Society of Anesthesiologists Task Force on Preanesthesia Evaluation. Anesthesiology 2012; 116:522–538.
  13. Institute for Clinical Systems Improvement (ICSI). Health care guideline: preoperative evaluation. 10th ed. Bloomington, MN: Institute for Clinical Systems Improvement; 2012.
  14. Johansson T, Fritsch G, Flamm M, et al. Effectiveness of non-cardiac preoperative testing in non-cardiac elective surgery: a systematic review. Br J Anaesth 2013; 110:926–939.
  15. Chung F, Yuan H, Yin L, Vairavanathan S, Wong DT. Elimination of preoperative testing in ambulatory surgery. Anesth Analg 2009; 108:467–475.
Issue
Cleveland Clinic Journal of Medicine - 82(10)
Issue
Cleveland Clinic Journal of Medicine - 82(10)
Page Number
664-666
Page Number
664-666
Publications
Publications
Topics
Article Type
Display Headline
Do healthy patients need routine laboratory testing before elective noncardiac surgery?
Display Headline
Do healthy patients need routine laboratory testing before elective noncardiac surgery?
Legacy Keywords
preoperative testing, Smart testing, guidelines, evidence-based medicine, defensive medicine, Nathan Houchens
Legacy Keywords
preoperative testing, Smart testing, guidelines, evidence-based medicine, defensive medicine, Nathan Houchens
Sections
Disallow All Ads
Alternative CME
Article PDF Media

Does this patient need ultrasonography of the leg to evaluate for deep vein thrombosis?

Article Type
Changed
Thu, 08/17/2017 - 13:25
Display Headline
Does this patient need ultrasonography of the leg to evaluate for deep vein thrombosis?

A 38-year-old woman presents to the emergency department after experiencing several days of swelling and mild discomfort in her left calf. She denies chest pain or shortness of breath. She does not recall antecedent trauma, is a nonsmoker, is healthy, and takes no medications apart from a multivitamin. She has not undergone any surgical procedure, has not been hospitalized recently, and has no history of venous thromboembolic disease. She says she started an aerobics program 1 week ago.

On examination, her left lower leg is mildly swollen, but the difference in calf circumference between the right and left legs is less than 1 cm. There is no erythema, no pitting edema, and only mild and rather diffuse tenderness of the calf. A urine pregnancy test is negative and her D-dimer level is 350 ng/mL (reference range < 500 ng/mL). Does she require ultrasonography of the left leg to evaluate for deep vein thrombosis (DVT)?

This patient does not need confirmatory ultrasonography, as her normal D-dimer level of 350 ng/mL is enough to rule out DVT. Her low probability of having DVT is further supported by her Wells score (Table 1), a tool that can help rule out DVT and reduce the need for further testing. DVT is unlikely if a patient’s Wells score is less than 2, and this patient’s score is –1. She receives 1 point for swelling of her left lower leg, but injury from her recent aerobic exercise is at least as likely as DVT to account for her symptoms (–2 points).

GUIDELINES AND CHOOSING WISELY

Compression ultrasonography is the study most commonly used to evaluate for DVT. The diagnosis is made if either the femoral or popliteal vein is noncompressible.1 In a patient with no history of DVT, the sensitivity of compression ultrasonography is 94%, and its specificity is 98%.

Several guidelines recommend using a clinical decision rule to establish the probability of venous thromboembolic disease before any additional diagnostic testing such as D-dimer measurement or ultrasonography.2–4 A number of clinical decision rules exist for DVT, but the Wells score is the most studied and validated.1 It incorporates the patient’s risk factors, symptoms, and signs to categorize the probability of DVT as low, moderate, or high and has been further modified to classify the risk as either likely or unlikely (Table 1).5

Guidelines from the American College of Chest Physicians (2012), Scottish Intercollegiate Guidelines Network (2010), and American Academy of Family Physicians and American College of Physicians (2007) recommend against performing imaging if a high-sensitivity D-dimer test is negative in a patient in whom the pretest probability of DVT is unlikely.2–4 Enzyme-linked immunofluorescence assays, microplate enzyme-linked immunosorbent assays, and latex quantitative assays are considered high-sensitivity D-dimer tests, having 96%, 94%, and 93% sensitivity, respectively, in ruling out DVT.1 Other D-dimer tests have lower sensitivity and cannot comfortably rule out DVT even if the results are negative.

Since D-dimer measurement is a sensitive but not specific test, it should be used only to rule out DVT—not to rule it in. Moreover, compression ultrasonography may be indicated to rule out other causes of the patient’s symptoms.

The guidelines caution against D-dimer testing if the patient has a comorbid condition that can by itself raise or lower the D-dimer level, leading one to falsely conclude the patient has or does not have DVT (Table 2).1–4 In these instances, the pretest probability of  DVT may be higher than calculated by a clinical prediction rule, and compression ultrasonography may be an appropriate initial test.4 Compression ultrasonography is also recommended as a confirmatory test in low-risk patients who have a positive D-dimer test or as an initial test in patients at higher risk for DVT.2–4

If a patient has a low pretest probability of DVT as defined by the Wells score and a normal high-sensitivity D-dimer measurement, then ordering imaging studies is a questionable practice according to statements by the American College of Physicians, American College of Emergency Physicians, European Society of Cardiology, American Academy of Family Physicians, and Scottish Intercollegiate Guidelines Network.

HARMS OF ULTRASONOGRAPHY

Although ultrasonography is generally well tolerated, it may be unnecessary. Combining a prediction rule (to assess the probability) with D-dimer testing (to rule out DVT) can significantly reduce the use of ultrasonography and the associated cost.

Wells et al5 calculated that clinicians could cut back on ultrasonographic testing by 39% by not doing it in those who had a low pretest probability and a negative D-dimer test result.5 In that patient population, fewer than 1% of patients were later found to have DVT.

Ordering compression ultrasonography as additional testing may lead to a false-positive result and to additional unnecessary testing and treatments that would inconvenience the patient, increase the risk of serious complications such as bleeding, and incur increased costs. Cost considerations should include not only the cost of the test and its interpretation, but also the workup and treatment of false-positive results, patient time missed from work while being tested, and potential associated costs for patients who need to be evaluated in the emergency department to obtain same-day testing.

THE CLINICAL BOTTOM LINE

Our patient’s Wells score indicates that DVT is unlikely. A negative D-dimer test is sufficient to rule out DVT, and further testing is unnecessary.

References
  1. Huisman MV, Klok FA. Diagnostic management of acute deep vein thrombosis and pulmonary embolism. J Thromb Haemost 2013; 11:412–422.
  2. Bates SM, Jaeschke R, Stevens EM, et al. Antithrombotic therapy and prevention of thrombosis, 9th edition: American College of Chest Physicians Evidence-Based Clinical Practice Guidelines. Chest 2012; 141(2 suppl):e351S–e418S.
  3. Scottish Intercollegiate Guidelines Network (SIGN). Prevention and management of venous thromboembolism. A national clinical guideline. Edinburgh (Scotland): Scottish Intercollegiate Guidelines Network (SIGN); 2010: http://sign.ac.uk/guidelines/fulltext/122/index.html. Accessed February 6, 2015.
  4. Qaseem A, Snow V, Barry P, et al. Current diagnosis of venous thromboembolism in primary care: a clinical practice guideline from the American Academy of Family Physicians and the American College of Physicians. Ann Intern Med 2007; 146:454–458.
  5. Wells PS, Anderson DR, Rodger M, et al. Evaluation of D-dimer in the diagnosis of suspected deep-vein thrombosis. N Engl J Med 2003; 349:1227–1235.
Article PDF
Author and Disclosure Information

C. Jessica Dine, MD, MSHPR
Assistant Professor of Medicine, Hospital of the University of Pennsylvania, Philadelphia

Sara L. Wallach, MD
St. Francis Medical Center, Trenton, NJ

Address: C. Jessica Dine, MD, MSHPR, Hospital of the University of Pennsylvania, Perelman Center for Advanced Medicine, 3400 Civic Center Boulevard, 1 West Pavilion, Philadelphia, PA 19104; e-mail: Jessica.Dine@uphs.upenn.edu

Smart Testing is a joint project of the Cleveland Clinic Journal of Medicine and the American College of Physicians (ACP). The series, an extension of the ACP High Value Care initiative (hvc.acponline.org/index.html), provides recommendations for improving patient outcomes while reducing unnecessary testing and treatment.

The views expressed in this article are those of the authors and do not necessarily reflect positions of the American College of Physicians.

Issue
Cleveland Clinic Journal of Medicine - 82(3)
Publications
Topics
Page Number
153-155
Legacy Keywords
deep vein thrombosis, DVT, venous thromboembolism, VTE, pulmonary embolism, PE, ultrasonography, ultrasound, Wells criteria, pretest probability, D-dimer, Jessica Dine, Sara Wallach
Sections
Author and Disclosure Information

C. Jessica Dine, MD, MSHPR
Assistant Professor of Medicine, Hospital of the University of Pennsylvania, Philadelphia

Sara L. Wallach, MD
St. Francis Medical Center, Trenton, NJ

Address: C. Jessica Dine, MD, MSHPR, Hospital of the University of Pennsylvania, Perelman Center for Advanced Medicine, 3400 Civic Center Boulevard, 1 West Pavilion, Philadelphia, PA 19104; e-mail: Jessica.Dine@uphs.upenn.edu

Smart Testing is a joint project of the Cleveland Clinic Journal of Medicine and the American College of Physicians (ACP). The series, an extension of the ACP High Value Care initiative (hvc.acponline.org/index.html), provides recommendations for improving patient outcomes while reducing unnecessary testing and treatment.

The views expressed in this article are those of the authors and do not necessarily reflect positions of the American College of Physicians.

Author and Disclosure Information

C. Jessica Dine, MD, MSHPR
Assistant Professor of Medicine, Hospital of the University of Pennsylvania, Philadelphia

Sara L. Wallach, MD
St. Francis Medical Center, Trenton, NJ

Address: C. Jessica Dine, MD, MSHPR, Hospital of the University of Pennsylvania, Perelman Center for Advanced Medicine, 3400 Civic Center Boulevard, 1 West Pavilion, Philadelphia, PA 19104; e-mail: Jessica.Dine@uphs.upenn.edu

Smart Testing is a joint project of the Cleveland Clinic Journal of Medicine and the American College of Physicians (ACP). The series, an extension of the ACP High Value Care initiative (hvc.acponline.org/index.html), provides recommendations for improving patient outcomes while reducing unnecessary testing and treatment.

The views expressed in this article are those of the authors and do not necessarily reflect positions of the American College of Physicians.

Article PDF
Article PDF
Related Articles

A 38-year-old woman presents to the emergency department after experiencing several days of swelling and mild discomfort in her left calf. She denies chest pain or shortness of breath. She does not recall antecedent trauma, is a nonsmoker, is healthy, and takes no medications apart from a multivitamin. She has not undergone any surgical procedure, has not been hospitalized recently, and has no history of venous thromboembolic disease. She says she started an aerobics program 1 week ago.

On examination, her left lower leg is mildly swollen, but the difference in calf circumference between the right and left legs is less than 1 cm. There is no erythema, no pitting edema, and only mild and rather diffuse tenderness of the calf. A urine pregnancy test is negative and her D-dimer level is 350 ng/mL (reference range < 500 ng/mL). Does she require ultrasonography of the left leg to evaluate for deep vein thrombosis (DVT)?

This patient does not need confirmatory ultrasonography, as her normal D-dimer level of 350 ng/mL is enough to rule out DVT. Her low probability of having DVT is further supported by her Wells score (Table 1), a tool that can help rule out DVT and reduce the need for further testing. DVT is unlikely if a patient’s Wells score is less than 2, and this patient’s score is –1. She receives 1 point for swelling of her left lower leg, but injury from her recent aerobic exercise is at least as likely as DVT to account for her symptoms (–2 points).

GUIDELINES AND CHOOSING WISELY

Compression ultrasonography is the study most commonly used to evaluate for DVT. The diagnosis is made if either the femoral or popliteal vein is noncompressible.1 In a patient with no history of DVT, the sensitivity of compression ultrasonography is 94%, and its specificity is 98%.

Several guidelines recommend using a clinical decision rule to establish the probability of venous thromboembolic disease before any additional diagnostic testing such as D-dimer measurement or ultrasonography.2–4 A number of clinical decision rules exist for DVT, but the Wells score is the most studied and validated.1 It incorporates the patient’s risk factors, symptoms, and signs to categorize the probability of DVT as low, moderate, or high and has been further modified to classify the risk as either likely or unlikely (Table 1).5

Guidelines from the American College of Chest Physicians (2012), Scottish Intercollegiate Guidelines Network (2010), and American Academy of Family Physicians and American College of Physicians (2007) recommend against performing imaging if a high-sensitivity D-dimer test is negative in a patient in whom the pretest probability of DVT is unlikely.2–4 Enzyme-linked immunofluorescence assays, microplate enzyme-linked immunosorbent assays, and latex quantitative assays are considered high-sensitivity D-dimer tests, having 96%, 94%, and 93% sensitivity, respectively, in ruling out DVT.1 Other D-dimer tests have lower sensitivity and cannot comfortably rule out DVT even if the results are negative.

Since D-dimer measurement is a sensitive but not specific test, it should be used only to rule out DVT—not to rule it in. Moreover, compression ultrasonography may be indicated to rule out other causes of the patient’s symptoms.

The guidelines caution against D-dimer testing if the patient has a comorbid condition that can by itself raise or lower the D-dimer level, leading one to falsely conclude the patient has or does not have DVT (Table 2).1–4 In these instances, the pretest probability of  DVT may be higher than calculated by a clinical prediction rule, and compression ultrasonography may be an appropriate initial test.4 Compression ultrasonography is also recommended as a confirmatory test in low-risk patients who have a positive D-dimer test or as an initial test in patients at higher risk for DVT.2–4

If a patient has a low pretest probability of DVT as defined by the Wells score and a normal high-sensitivity D-dimer measurement, then ordering imaging studies is a questionable practice according to statements by the American College of Physicians, American College of Emergency Physicians, European Society of Cardiology, American Academy of Family Physicians, and Scottish Intercollegiate Guidelines Network.

HARMS OF ULTRASONOGRAPHY

Although ultrasonography is generally well tolerated, it may be unnecessary. Combining a prediction rule (to assess the probability) with D-dimer testing (to rule out DVT) can significantly reduce the use of ultrasonography and the associated cost.

Wells et al5 calculated that clinicians could cut back on ultrasonographic testing by 39% by not doing it in those who had a low pretest probability and a negative D-dimer test result.5 In that patient population, fewer than 1% of patients were later found to have DVT.

Ordering compression ultrasonography as additional testing may lead to a false-positive result and to additional unnecessary testing and treatments that would inconvenience the patient, increase the risk of serious complications such as bleeding, and incur increased costs. Cost considerations should include not only the cost of the test and its interpretation, but also the workup and treatment of false-positive results, patient time missed from work while being tested, and potential associated costs for patients who need to be evaluated in the emergency department to obtain same-day testing.

THE CLINICAL BOTTOM LINE

Our patient’s Wells score indicates that DVT is unlikely. A negative D-dimer test is sufficient to rule out DVT, and further testing is unnecessary.

A 38-year-old woman presents to the emergency department after experiencing several days of swelling and mild discomfort in her left calf. She denies chest pain or shortness of breath. She does not recall antecedent trauma, is a nonsmoker, is healthy, and takes no medications apart from a multivitamin. She has not undergone any surgical procedure, has not been hospitalized recently, and has no history of venous thromboembolic disease. She says she started an aerobics program 1 week ago.

On examination, her left lower leg is mildly swollen, but the difference in calf circumference between the right and left legs is less than 1 cm. There is no erythema, no pitting edema, and only mild and rather diffuse tenderness of the calf. A urine pregnancy test is negative and her D-dimer level is 350 ng/mL (reference range < 500 ng/mL). Does she require ultrasonography of the left leg to evaluate for deep vein thrombosis (DVT)?

This patient does not need confirmatory ultrasonography, as her normal D-dimer level of 350 ng/mL is enough to rule out DVT. Her low probability of having DVT is further supported by her Wells score (Table 1), a tool that can help rule out DVT and reduce the need for further testing. DVT is unlikely if a patient’s Wells score is less than 2, and this patient’s score is –1. She receives 1 point for swelling of her left lower leg, but injury from her recent aerobic exercise is at least as likely as DVT to account for her symptoms (–2 points).

GUIDELINES AND CHOOSING WISELY

Compression ultrasonography is the study most commonly used to evaluate for DVT. The diagnosis is made if either the femoral or popliteal vein is noncompressible.1 In a patient with no history of DVT, the sensitivity of compression ultrasonography is 94%, and its specificity is 98%.

Several guidelines recommend using a clinical decision rule to establish the probability of venous thromboembolic disease before any additional diagnostic testing such as D-dimer measurement or ultrasonography.2–4 A number of clinical decision rules exist for DVT, but the Wells score is the most studied and validated.1 It incorporates the patient’s risk factors, symptoms, and signs to categorize the probability of DVT as low, moderate, or high and has been further modified to classify the risk as either likely or unlikely (Table 1).5

Guidelines from the American College of Chest Physicians (2012), Scottish Intercollegiate Guidelines Network (2010), and American Academy of Family Physicians and American College of Physicians (2007) recommend against performing imaging if a high-sensitivity D-dimer test is negative in a patient in whom the pretest probability of DVT is unlikely.2–4 Enzyme-linked immunofluorescence assays, microplate enzyme-linked immunosorbent assays, and latex quantitative assays are considered high-sensitivity D-dimer tests, having 96%, 94%, and 93% sensitivity, respectively, in ruling out DVT.1 Other D-dimer tests have lower sensitivity and cannot comfortably rule out DVT even if the results are negative.

Since D-dimer measurement is a sensitive but not specific test, it should be used only to rule out DVT—not to rule it in. Moreover, compression ultrasonography may be indicated to rule out other causes of the patient’s symptoms.

The guidelines caution against D-dimer testing if the patient has a comorbid condition that can by itself raise or lower the D-dimer level, leading one to falsely conclude the patient has or does not have DVT (Table 2).1–4 In these instances, the pretest probability of  DVT may be higher than calculated by a clinical prediction rule, and compression ultrasonography may be an appropriate initial test.4 Compression ultrasonography is also recommended as a confirmatory test in low-risk patients who have a positive D-dimer test or as an initial test in patients at higher risk for DVT.2–4

If a patient has a low pretest probability of DVT as defined by the Wells score and a normal high-sensitivity D-dimer measurement, then ordering imaging studies is a questionable practice according to statements by the American College of Physicians, American College of Emergency Physicians, European Society of Cardiology, American Academy of Family Physicians, and Scottish Intercollegiate Guidelines Network.

HARMS OF ULTRASONOGRAPHY

Although ultrasonography is generally well tolerated, it may be unnecessary. Combining a prediction rule (to assess the probability) with D-dimer testing (to rule out DVT) can significantly reduce the use of ultrasonography and the associated cost.

Wells et al5 calculated that clinicians could cut back on ultrasonographic testing by 39% by not doing it in those who had a low pretest probability and a negative D-dimer test result.5 In that patient population, fewer than 1% of patients were later found to have DVT.

Ordering compression ultrasonography as additional testing may lead to a false-positive result and to additional unnecessary testing and treatments that would inconvenience the patient, increase the risk of serious complications such as bleeding, and incur increased costs. Cost considerations should include not only the cost of the test and its interpretation, but also the workup and treatment of false-positive results, patient time missed from work while being tested, and potential associated costs for patients who need to be evaluated in the emergency department to obtain same-day testing.

THE CLINICAL BOTTOM LINE

Our patient’s Wells score indicates that DVT is unlikely. A negative D-dimer test is sufficient to rule out DVT, and further testing is unnecessary.

References
  1. Huisman MV, Klok FA. Diagnostic management of acute deep vein thrombosis and pulmonary embolism. J Thromb Haemost 2013; 11:412–422.
  2. Bates SM, Jaeschke R, Stevens EM, et al. Antithrombotic therapy and prevention of thrombosis, 9th edition: American College of Chest Physicians Evidence-Based Clinical Practice Guidelines. Chest 2012; 141(2 suppl):e351S–e418S.
  3. Scottish Intercollegiate Guidelines Network (SIGN). Prevention and management of venous thromboembolism. A national clinical guideline. Edinburgh (Scotland): Scottish Intercollegiate Guidelines Network (SIGN); 2010: http://sign.ac.uk/guidelines/fulltext/122/index.html. Accessed February 6, 2015.
  4. Qaseem A, Snow V, Barry P, et al. Current diagnosis of venous thromboembolism in primary care: a clinical practice guideline from the American Academy of Family Physicians and the American College of Physicians. Ann Intern Med 2007; 146:454–458.
  5. Wells PS, Anderson DR, Rodger M, et al. Evaluation of D-dimer in the diagnosis of suspected deep-vein thrombosis. N Engl J Med 2003; 349:1227–1235.
References
  1. Huisman MV, Klok FA. Diagnostic management of acute deep vein thrombosis and pulmonary embolism. J Thromb Haemost 2013; 11:412–422.
  2. Bates SM, Jaeschke R, Stevens EM, et al. Antithrombotic therapy and prevention of thrombosis, 9th edition: American College of Chest Physicians Evidence-Based Clinical Practice Guidelines. Chest 2012; 141(2 suppl):e351S–e418S.
  3. Scottish Intercollegiate Guidelines Network (SIGN). Prevention and management of venous thromboembolism. A national clinical guideline. Edinburgh (Scotland): Scottish Intercollegiate Guidelines Network (SIGN); 2010: http://sign.ac.uk/guidelines/fulltext/122/index.html. Accessed February 6, 2015.
  4. Qaseem A, Snow V, Barry P, et al. Current diagnosis of venous thromboembolism in primary care: a clinical practice guideline from the American Academy of Family Physicians and the American College of Physicians. Ann Intern Med 2007; 146:454–458.
  5. Wells PS, Anderson DR, Rodger M, et al. Evaluation of D-dimer in the diagnosis of suspected deep-vein thrombosis. N Engl J Med 2003; 349:1227–1235.
Issue
Cleveland Clinic Journal of Medicine - 82(3)
Issue
Cleveland Clinic Journal of Medicine - 82(3)
Page Number
153-155
Page Number
153-155
Publications
Publications
Topics
Article Type
Display Headline
Does this patient need ultrasonography of the leg to evaluate for deep vein thrombosis?
Display Headline
Does this patient need ultrasonography of the leg to evaluate for deep vein thrombosis?
Legacy Keywords
deep vein thrombosis, DVT, venous thromboembolism, VTE, pulmonary embolism, PE, ultrasonography, ultrasound, Wells criteria, pretest probability, D-dimer, Jessica Dine, Sara Wallach
Legacy Keywords
deep vein thrombosis, DVT, venous thromboembolism, VTE, pulmonary embolism, PE, ultrasonography, ultrasound, Wells criteria, pretest probability, D-dimer, Jessica Dine, Sara Wallach
Sections
Disallow All Ads
Alternative CME
Article PDF Media

When does an adult with headaches need central nervous system imaging?

Article Type
Changed
Thu, 09/14/2017 - 12:19
Display Headline
When does an adult with headaches need central nervous system imaging?

A 32-year-old woman presents to the clinic  for evaluation of headaches, which she describes as pulsatile and throbbing, usually unilateral but involving different sides of the head at different times, and severe, causing her to miss work. They usually last between 12 and 24 hours and are associated with nausea but no vomiting and no changes in vision. They are worse around the time of her menses, have been occurring about twice a month for the past 6 months, and respond to ibuprofen. She thought they were caused by chronic seasonal allergies and sinusitis and has tried antihistamines and nasal irrigation without success. They are not affected by body position, they are not explosive, and they are not brought on by the Valsalva maneuver. She reports no other neurologic or systemic symptoms.

A detailed neurologic examination shows no deficits. However, the patient is concerned, as one of her friends was recently diagnosed with cancer. She requests imaging to “make sure there is no cancer.” Would it be appropriate to order imaging at this time?

No, it would not. Patients who have primary headache disorders without red-flag symptoms should not undergo imaging of the central nervous system (CNS) as part of their initial evaluation.1–4 (The list of potential red-flag symptoms is long but includes new onset after age 50, persistent neurologic changes, systemic symptoms or immunosuppression, sudden onset, progressive pain, positional nature, headaches precipitated by the Valsalva maneuver, and papilledema.)

Without red-flag symptoms, CNS imaging is unwarranted and may be harmful

CNS imaging may be appropriate for patients with features that increase the likelihood of structural diseases such as arteriovenous malformation, aneurysm, tumor, or subarachnoid hemorrhage. This patient, however, does not have worrisome signs or symptoms. Her symptoms are most consistent with migraine headache without aura. In patients with migraine headache without symptoms suggesting structural disease, CNS imaging is unwarranted and may be harmful.

DIAGNOSING MIGRAINE ACCURATELY

Diagnosing migraine headache can be a challenge, and up to half of all patients with migraine may be undiagnosed.5 The proper diagnosis of headache type is critical to the initial evaluation. In diagnosing migraine, one can use the mnemonic POUND4:

  • Pulsatile
  • One-day duration (4–72 hours)
  • Unilateral
  • Nausea or vomiting
  • Disabling.

If four or five of these features are present, the likelihood ratio that the patient has migraine headache is 24, making it overwhelmingly likely that is the correct diagnosis.4 With three features the likelihood ratio is 3.5. If two or fewer features are present, migraine is much less likely, with a likelihood ratio of 0.41. Thus, patients with classic symptoms of migraine can be confidently and accurately diagnosed without the need for any imaging studies. 

The patient in the vignette has all five POUND criteria. If we estimate her pretest probability of migraine headache at 50% (which is actually a conservative estimate—see Guidelines and Choosing Wisely, below), then, utilizing Bayes’ theorem, the likelihood ratio of 24 would result in a 95% probability that her headaches represent migraine.

 

 

GUIDELINES AND CHOOSING WISELY

High-quality reviews have found no benefit in performing imaging for primary headache disorders.1–3 This is due, in large part, to the rarity of secondary headache disorders in the primary care setting. In fact, most patients—90% in one study6—presenting to their primary care physicians with headaches meet the diagnostic criteria for migraine.

When CNS imaging is needed, consider MRI instead of CT

Significant abnormalities on imaging in patients with migraine headaches are also very rare. In patients with migraine headaches who undergo imaging, the rate of worrisome abnormalities that could lead to a change in management (0.2%) is less than that in the general population at the time of autopsy (0.8%).7

As part of the Choosing Wisely campaign, the American College of Radiology and the American Headache Society recommend against imaging for patients at low risk with migraine headaches. Because of the potential for harm from radiation exposure, the American Headache Society also recommends against computed tomography (CT) for evaluating headaches when magnetic resonance imaging (MRI) is available, except in emergencies.

Lists of tests and treatments that physicians and patients should question and discuss together to make wise decisions are available at www.choosingwisely.org.

HARMS ASSOCIATED WITH CNS IMAGING

Medical tests can be associated with significant harm. Potential harms of head imaging include radiation exposure from CT and false-positive findings. These false-positives, such as the finding of lesions that eventually prove to be benign, may require further testing and cause significant anxiety to the patient. 

The effective radiation dose from a CT scan of the head is 2.0 mSv, equivalent to 250 days of background radiation exposure or 100 chest radiographs. Radiation exposure has been linked to increased risk of fatal cancer, and the risks increase with subsequent radiation doses.8

Incidental findings are common on head imaging and often lead to additional medical procedures and workup, without improvements in patient well-being. While the harms of false-positive testing and the finding of benign lesions are difficult to quantify, it is clear that downstream costs can accumulate and that these results cause significant undue worry to the patient.

CLINICAL BOTTOM LINE

Patients with migraine headache who do not have red-flag signs or symptoms are unlikely to benefit from CNS imaging and may experience harm. The rate of abnormalities in this population is not significantly different from that in the general population. A thorough history and physical examination should be done to find the proper diagnosis and to uncover any red-flag symptoms. For migraine headaches that are worsened by identified triggers, those triggers should be addressed before further evaluation is performed. When imaging is needed, physicians should consider minimizing radiation risk by ordering MRI instead of CT.

References
  1. Beithon J, Gallenberg M, Johnson K, et al; Institute for Clinical Systems Improvement. Diagnosis and treatment of headache. www.icsi.org/_asset/qwrznq/headache.pdf. Accessed September 5, 2014.
  2. Frishberg BM, Rosenberg JH, Matchar DB, et al; US Headache Consortium. Evidence-based guidelines in the primary care setting: neuroimaging in patients with nonacute headache. www.aan.com/professionals/practice/pdfs/gl0088.pdf. Accessed September 5, 2014.
  3. Silberstein SD. Practice parameter: evidence-based guidelines for migraine headache (an evidence-based review): report of the Quality Standards Subcommittee of the American Academy of Neurology. Neurology 2000; 55:754–762.
  4. Detsky ME, McDonald DR, Baerlocher MO, Tomlinson GA, McCrory DC, Booth CM. Does this patient with headache have a migraine or need neuroimaging? JAMA 2006; 296:1274–1283.
  5. Lipton RB, Diamond S, Reed M, Diamond ML, Stewart WF. Migraine diagnosis and treatment: results from the American Migraine Study II. Headache 2001; 41:638–645.
  6. Dowson A, Dahlof C, Tepper S, Newman L. Prevalence and diagnosis of migraine in a primary care setting (abstract). Cephalalgia 2002; 22:590–591.
  7. Frishberg BM. The utility of neuroimaging in the evaluation of headache in patients with normal neurologic examinations. Neurology 1994; 44:1191–1197.
  8. Semelka RC, Armao DM, Elias J Jr, Huda W. Imaging strategies to reduce the risk of radiation in CT studies, including selective substitution with MRI. J Magn Reson Imaging 2007; 25:900–909.
Article PDF
Author and Disclosure Information

Jason Post, MD
Assistant Professor of Medicine, Department of Medicine, Mayo Clinic, Rochester, MN

Christopher Moriates, MD
Assistant Clinical Professor, Division of Hospital Medicine, University of California, San Francisco; Director of Caring Wisely Program, Center for Healthcare Value, University of California, San Francisco

Address: Jason Post, MD, Department of Medicine, 200 First Street SW, Mayo Clinic, Rochester, MN 55905; e-mail: Post.Jason@mayo.edu

Smart Testing is a joint project of the Cleveland Clinic Journal of Medicine and the American College of Physicians (ACP). The series, an extension of the ACP High Value Care initiative (hvc.acponline.org/index.html), provides recommendations for improving patient outcomes while reducing unnecessary testing and treatment.

The views expressed in this article are those of the authors and do not necessarily reflect positions of the American College of Physicians.

Issue
Cleveland Clinic Journal of Medicine - 81(12)
Publications
Topics
Page Number
719-720
Legacy Keywords
headache, migraine, imaging, magnetic resonance imaging, MRI, computed tomography, CT, Jason Post, Christopher Moriates
Sections
Author and Disclosure Information

Jason Post, MD
Assistant Professor of Medicine, Department of Medicine, Mayo Clinic, Rochester, MN

Christopher Moriates, MD
Assistant Clinical Professor, Division of Hospital Medicine, University of California, San Francisco; Director of Caring Wisely Program, Center for Healthcare Value, University of California, San Francisco

Address: Jason Post, MD, Department of Medicine, 200 First Street SW, Mayo Clinic, Rochester, MN 55905; e-mail: Post.Jason@mayo.edu

Smart Testing is a joint project of the Cleveland Clinic Journal of Medicine and the American College of Physicians (ACP). The series, an extension of the ACP High Value Care initiative (hvc.acponline.org/index.html), provides recommendations for improving patient outcomes while reducing unnecessary testing and treatment.

The views expressed in this article are those of the authors and do not necessarily reflect positions of the American College of Physicians.

Author and Disclosure Information

Jason Post, MD
Assistant Professor of Medicine, Department of Medicine, Mayo Clinic, Rochester, MN

Christopher Moriates, MD
Assistant Clinical Professor, Division of Hospital Medicine, University of California, San Francisco; Director of Caring Wisely Program, Center for Healthcare Value, University of California, San Francisco

Address: Jason Post, MD, Department of Medicine, 200 First Street SW, Mayo Clinic, Rochester, MN 55905; e-mail: Post.Jason@mayo.edu

Smart Testing is a joint project of the Cleveland Clinic Journal of Medicine and the American College of Physicians (ACP). The series, an extension of the ACP High Value Care initiative (hvc.acponline.org/index.html), provides recommendations for improving patient outcomes while reducing unnecessary testing and treatment.

The views expressed in this article are those of the authors and do not necessarily reflect positions of the American College of Physicians.

Article PDF
Article PDF
Related Articles

A 32-year-old woman presents to the clinic  for evaluation of headaches, which she describes as pulsatile and throbbing, usually unilateral but involving different sides of the head at different times, and severe, causing her to miss work. They usually last between 12 and 24 hours and are associated with nausea but no vomiting and no changes in vision. They are worse around the time of her menses, have been occurring about twice a month for the past 6 months, and respond to ibuprofen. She thought they were caused by chronic seasonal allergies and sinusitis and has tried antihistamines and nasal irrigation without success. They are not affected by body position, they are not explosive, and they are not brought on by the Valsalva maneuver. She reports no other neurologic or systemic symptoms.

A detailed neurologic examination shows no deficits. However, the patient is concerned, as one of her friends was recently diagnosed with cancer. She requests imaging to “make sure there is no cancer.” Would it be appropriate to order imaging at this time?

No, it would not. Patients who have primary headache disorders without red-flag symptoms should not undergo imaging of the central nervous system (CNS) as part of their initial evaluation.1–4 (The list of potential red-flag symptoms is long but includes new onset after age 50, persistent neurologic changes, systemic symptoms or immunosuppression, sudden onset, progressive pain, positional nature, headaches precipitated by the Valsalva maneuver, and papilledema.)

Without red-flag symptoms, CNS imaging is unwarranted and may be harmful

CNS imaging may be appropriate for patients with features that increase the likelihood of structural diseases such as arteriovenous malformation, aneurysm, tumor, or subarachnoid hemorrhage. This patient, however, does not have worrisome signs or symptoms. Her symptoms are most consistent with migraine headache without aura. In patients with migraine headache without symptoms suggesting structural disease, CNS imaging is unwarranted and may be harmful.

DIAGNOSING MIGRAINE ACCURATELY

Diagnosing migraine headache can be a challenge, and up to half of all patients with migraine may be undiagnosed.5 The proper diagnosis of headache type is critical to the initial evaluation. In diagnosing migraine, one can use the mnemonic POUND4:

  • Pulsatile
  • One-day duration (4–72 hours)
  • Unilateral
  • Nausea or vomiting
  • Disabling.

If four or five of these features are present, the likelihood ratio that the patient has migraine headache is 24, making it overwhelmingly likely that is the correct diagnosis.4 With three features the likelihood ratio is 3.5. If two or fewer features are present, migraine is much less likely, with a likelihood ratio of 0.41. Thus, patients with classic symptoms of migraine can be confidently and accurately diagnosed without the need for any imaging studies. 

The patient in the vignette has all five POUND criteria. If we estimate her pretest probability of migraine headache at 50% (which is actually a conservative estimate—see Guidelines and Choosing Wisely, below), then, utilizing Bayes’ theorem, the likelihood ratio of 24 would result in a 95% probability that her headaches represent migraine.

 

 

GUIDELINES AND CHOOSING WISELY

High-quality reviews have found no benefit in performing imaging for primary headache disorders.1–3 This is due, in large part, to the rarity of secondary headache disorders in the primary care setting. In fact, most patients—90% in one study6—presenting to their primary care physicians with headaches meet the diagnostic criteria for migraine.

When CNS imaging is needed, consider MRI instead of CT

Significant abnormalities on imaging in patients with migraine headaches are also very rare. In patients with migraine headaches who undergo imaging, the rate of worrisome abnormalities that could lead to a change in management (0.2%) is less than that in the general population at the time of autopsy (0.8%).7

As part of the Choosing Wisely campaign, the American College of Radiology and the American Headache Society recommend against imaging for patients at low risk with migraine headaches. Because of the potential for harm from radiation exposure, the American Headache Society also recommends against computed tomography (CT) for evaluating headaches when magnetic resonance imaging (MRI) is available, except in emergencies.

Lists of tests and treatments that physicians and patients should question and discuss together to make wise decisions are available at www.choosingwisely.org.

HARMS ASSOCIATED WITH CNS IMAGING

Medical tests can be associated with significant harm. Potential harms of head imaging include radiation exposure from CT and false-positive findings. These false-positives, such as the finding of lesions that eventually prove to be benign, may require further testing and cause significant anxiety to the patient. 

The effective radiation dose from a CT scan of the head is 2.0 mSv, equivalent to 250 days of background radiation exposure or 100 chest radiographs. Radiation exposure has been linked to increased risk of fatal cancer, and the risks increase with subsequent radiation doses.8

Incidental findings are common on head imaging and often lead to additional medical procedures and workup, without improvements in patient well-being. While the harms of false-positive testing and the finding of benign lesions are difficult to quantify, it is clear that downstream costs can accumulate and that these results cause significant undue worry to the patient.

CLINICAL BOTTOM LINE

Patients with migraine headache who do not have red-flag signs or symptoms are unlikely to benefit from CNS imaging and may experience harm. The rate of abnormalities in this population is not significantly different from that in the general population. A thorough history and physical examination should be done to find the proper diagnosis and to uncover any red-flag symptoms. For migraine headaches that are worsened by identified triggers, those triggers should be addressed before further evaluation is performed. When imaging is needed, physicians should consider minimizing radiation risk by ordering MRI instead of CT.

A 32-year-old woman presents to the clinic  for evaluation of headaches, which she describes as pulsatile and throbbing, usually unilateral but involving different sides of the head at different times, and severe, causing her to miss work. They usually last between 12 and 24 hours and are associated with nausea but no vomiting and no changes in vision. They are worse around the time of her menses, have been occurring about twice a month for the past 6 months, and respond to ibuprofen. She thought they were caused by chronic seasonal allergies and sinusitis and has tried antihistamines and nasal irrigation without success. They are not affected by body position, they are not explosive, and they are not brought on by the Valsalva maneuver. She reports no other neurologic or systemic symptoms.

A detailed neurologic examination shows no deficits. However, the patient is concerned, as one of her friends was recently diagnosed with cancer. She requests imaging to “make sure there is no cancer.” Would it be appropriate to order imaging at this time?

No, it would not. Patients who have primary headache disorders without red-flag symptoms should not undergo imaging of the central nervous system (CNS) as part of their initial evaluation.1–4 (The list of potential red-flag symptoms is long but includes new onset after age 50, persistent neurologic changes, systemic symptoms or immunosuppression, sudden onset, progressive pain, positional nature, headaches precipitated by the Valsalva maneuver, and papilledema.)

Without red-flag symptoms, CNS imaging is unwarranted and may be harmful

CNS imaging may be appropriate for patients with features that increase the likelihood of structural diseases such as arteriovenous malformation, aneurysm, tumor, or subarachnoid hemorrhage. This patient, however, does not have worrisome signs or symptoms. Her symptoms are most consistent with migraine headache without aura. In patients with migraine headache without symptoms suggesting structural disease, CNS imaging is unwarranted and may be harmful.

DIAGNOSING MIGRAINE ACCURATELY

Diagnosing migraine headache can be a challenge, and up to half of all patients with migraine may be undiagnosed.5 The proper diagnosis of headache type is critical to the initial evaluation. In diagnosing migraine, one can use the mnemonic POUND4:

  • Pulsatile
  • One-day duration (4–72 hours)
  • Unilateral
  • Nausea or vomiting
  • Disabling.

If four or five of these features are present, the likelihood ratio that the patient has migraine headache is 24, making it overwhelmingly likely that is the correct diagnosis.4 With three features the likelihood ratio is 3.5. If two or fewer features are present, migraine is much less likely, with a likelihood ratio of 0.41. Thus, patients with classic symptoms of migraine can be confidently and accurately diagnosed without the need for any imaging studies. 

The patient in the vignette has all five POUND criteria. If we estimate her pretest probability of migraine headache at 50% (which is actually a conservative estimate—see Guidelines and Choosing Wisely, below), then, utilizing Bayes’ theorem, the likelihood ratio of 24 would result in a 95% probability that her headaches represent migraine.

 

 

GUIDELINES AND CHOOSING WISELY

High-quality reviews have found no benefit in performing imaging for primary headache disorders.1–3 This is due, in large part, to the rarity of secondary headache disorders in the primary care setting. In fact, most patients—90% in one study6—presenting to their primary care physicians with headaches meet the diagnostic criteria for migraine.

When CNS imaging is needed, consider MRI instead of CT

Significant abnormalities on imaging in patients with migraine headaches are also very rare. In patients with migraine headaches who undergo imaging, the rate of worrisome abnormalities that could lead to a change in management (0.2%) is less than that in the general population at the time of autopsy (0.8%).7

As part of the Choosing Wisely campaign, the American College of Radiology and the American Headache Society recommend against imaging for patients at low risk with migraine headaches. Because of the potential for harm from radiation exposure, the American Headache Society also recommends against computed tomography (CT) for evaluating headaches when magnetic resonance imaging (MRI) is available, except in emergencies.

Lists of tests and treatments that physicians and patients should question and discuss together to make wise decisions are available at www.choosingwisely.org.

HARMS ASSOCIATED WITH CNS IMAGING

Medical tests can be associated with significant harm. Potential harms of head imaging include radiation exposure from CT and false-positive findings. These false-positives, such as the finding of lesions that eventually prove to be benign, may require further testing and cause significant anxiety to the patient. 

The effective radiation dose from a CT scan of the head is 2.0 mSv, equivalent to 250 days of background radiation exposure or 100 chest radiographs. Radiation exposure has been linked to increased risk of fatal cancer, and the risks increase with subsequent radiation doses.8

Incidental findings are common on head imaging and often lead to additional medical procedures and workup, without improvements in patient well-being. While the harms of false-positive testing and the finding of benign lesions are difficult to quantify, it is clear that downstream costs can accumulate and that these results cause significant undue worry to the patient.

CLINICAL BOTTOM LINE

Patients with migraine headache who do not have red-flag signs or symptoms are unlikely to benefit from CNS imaging and may experience harm. The rate of abnormalities in this population is not significantly different from that in the general population. A thorough history and physical examination should be done to find the proper diagnosis and to uncover any red-flag symptoms. For migraine headaches that are worsened by identified triggers, those triggers should be addressed before further evaluation is performed. When imaging is needed, physicians should consider minimizing radiation risk by ordering MRI instead of CT.

References
  1. Beithon J, Gallenberg M, Johnson K, et al; Institute for Clinical Systems Improvement. Diagnosis and treatment of headache. www.icsi.org/_asset/qwrznq/headache.pdf. Accessed September 5, 2014.
  2. Frishberg BM, Rosenberg JH, Matchar DB, et al; US Headache Consortium. Evidence-based guidelines in the primary care setting: neuroimaging in patients with nonacute headache. www.aan.com/professionals/practice/pdfs/gl0088.pdf. Accessed September 5, 2014.
  3. Silberstein SD. Practice parameter: evidence-based guidelines for migraine headache (an evidence-based review): report of the Quality Standards Subcommittee of the American Academy of Neurology. Neurology 2000; 55:754–762.
  4. Detsky ME, McDonald DR, Baerlocher MO, Tomlinson GA, McCrory DC, Booth CM. Does this patient with headache have a migraine or need neuroimaging? JAMA 2006; 296:1274–1283.
  5. Lipton RB, Diamond S, Reed M, Diamond ML, Stewart WF. Migraine diagnosis and treatment: results from the American Migraine Study II. Headache 2001; 41:638–645.
  6. Dowson A, Dahlof C, Tepper S, Newman L. Prevalence and diagnosis of migraine in a primary care setting (abstract). Cephalalgia 2002; 22:590–591.
  7. Frishberg BM. The utility of neuroimaging in the evaluation of headache in patients with normal neurologic examinations. Neurology 1994; 44:1191–1197.
  8. Semelka RC, Armao DM, Elias J Jr, Huda W. Imaging strategies to reduce the risk of radiation in CT studies, including selective substitution with MRI. J Magn Reson Imaging 2007; 25:900–909.
References
  1. Beithon J, Gallenberg M, Johnson K, et al; Institute for Clinical Systems Improvement. Diagnosis and treatment of headache. www.icsi.org/_asset/qwrznq/headache.pdf. Accessed September 5, 2014.
  2. Frishberg BM, Rosenberg JH, Matchar DB, et al; US Headache Consortium. Evidence-based guidelines in the primary care setting: neuroimaging in patients with nonacute headache. www.aan.com/professionals/practice/pdfs/gl0088.pdf. Accessed September 5, 2014.
  3. Silberstein SD. Practice parameter: evidence-based guidelines for migraine headache (an evidence-based review): report of the Quality Standards Subcommittee of the American Academy of Neurology. Neurology 2000; 55:754–762.
  4. Detsky ME, McDonald DR, Baerlocher MO, Tomlinson GA, McCrory DC, Booth CM. Does this patient with headache have a migraine or need neuroimaging? JAMA 2006; 296:1274–1283.
  5. Lipton RB, Diamond S, Reed M, Diamond ML, Stewart WF. Migraine diagnosis and treatment: results from the American Migraine Study II. Headache 2001; 41:638–645.
  6. Dowson A, Dahlof C, Tepper S, Newman L. Prevalence and diagnosis of migraine in a primary care setting (abstract). Cephalalgia 2002; 22:590–591.
  7. Frishberg BM. The utility of neuroimaging in the evaluation of headache in patients with normal neurologic examinations. Neurology 1994; 44:1191–1197.
  8. Semelka RC, Armao DM, Elias J Jr, Huda W. Imaging strategies to reduce the risk of radiation in CT studies, including selective substitution with MRI. J Magn Reson Imaging 2007; 25:900–909.
Issue
Cleveland Clinic Journal of Medicine - 81(12)
Issue
Cleveland Clinic Journal of Medicine - 81(12)
Page Number
719-720
Page Number
719-720
Publications
Publications
Topics
Article Type
Display Headline
When does an adult with headaches need central nervous system imaging?
Display Headline
When does an adult with headaches need central nervous system imaging?
Legacy Keywords
headache, migraine, imaging, magnetic resonance imaging, MRI, computed tomography, CT, Jason Post, Christopher Moriates
Legacy Keywords
headache, migraine, imaging, magnetic resonance imaging, MRI, computed tomography, CT, Jason Post, Christopher Moriates
Sections
Disallow All Ads
Alternative CME
Article PDF Media

Should all patients have a resting 12-lead ECG before elective noncardiac surgery?

Article Type
Changed
Wed, 09/13/2017 - 15:44
Display Headline
Should all patients have a resting 12-lead ECG before elective noncardiac surgery?

A 55-year-old lawyer with hypertension well controlled on lisinopril and amlodipine is scheduled for elective hernia repair under general anesthesia. His surgeon has referred him for a preoperative evaluation. He has never smoked, runs 4 miles on days off from work, and enjoys long hiking trips. On examination, his body mass index is 26 kg/m2 and his blood pressure is 130/78 mm Hg; his cardiac examination and the rest of the clinical examination are unremarkable. He asks if he should have an electrocardiogram (ECG) as a part of his workup.

A preoperative ECG is not routinely recommended in all asymptomatic patients undergoing noncardiac surgery.

Consider obtaining an ECG in patients planning to undergo a high-risk surgical procedure, especially if they have one or more clinical risk factors for coronary artery disease, and in patients undergoing elevated-cardiac-risk surgery who are known to have coronary artery disease, chronic heart failure, peripheral arterial disease, or cerebrovascular disease. However, a preoperative ECG is not routinely recommended for patients perceived to be at low cardiac risk who are planning to undergo low-risk surgery. In those patients it could delay surgery unnecessarily, cause further unnecessary testing, drive up costs, and increase patient anxiety.

Here we discuss the perioperative cardiac risk based on type of surgery and patient characteristics and summarize the current guidelines and recommendations on obtaining a preoperative 12-lead ECG in patients undergoing noncardiac surgery.

RISK DEPENDS ON TYPE OF SURGERY AND PATIENT FACTORS

Physicians, including primary care physicians, hospitalists, cardiologists, and anesthesiologists, are routinely asked to evaluate patients before surgical procedures. The purpose of the preoperative evaluation is to optimize existing medical conditions, to identify undiagnosed conditions that can increase risk of perioperative morbidity and death, and to suggest strategies to mitigate risk.1,2

Cardiac risk is multifactorial, and risk factors for postoperative adverse cardiac events include the type of surgery and patient factors.1,3

Cardiac risk based on type of surgery

Low-risk procedures are those in which the risk of a perioperative major adverse cardiac event is less than 1%.1,4 Examples:

  • Ambulatory surgery
  • Breast or plastic surgery
  • Cataract surgery
  • Endoscopic procedures.

Elevated-risk procedures are those in which the risk is 1% or higher. Examples:

  • Intraperitoneal surgery
  • Intrathoracic surgery
  • Carotid endarterectomy
  • Head and neck surgery
  • Orthopedic surgery
  • Prostate surgery
  • Aortic surgery
  • Major vascular surgery
  • Peripheral arterial surgery.

Cardiac risk based on patient factors

The 2014 American College of Cardiology and American Heart Association (ACC/AHA) perioperative guidelines list a number of clinical risk factors for perioperative cardiac morbidity and death.1 These include coronary artery disease, chronic heart failure, clinically suspected moderate or greater degrees of valvular heart disease, arrhythmias, conduction disorders, pulmonary vascular disease, and adult congenital heart disease.

Patients with these conditions and patients with unstable coronary syndromes warrant preoperative ECGs and sometimes even urgent interventions before any nonemergency surgery, provided such interventions would affect decision-making and perioperative care.1

The risk of perioperative cardiac morbidity and death can be calculated using either the Revised Cardiac Risk Index scoring system or the American College of Surgeons National Surgical Quality Improvement Program calculator.157 The former is fairly simple, validated, and accepted, while the latter requires use of online calculators (eg, www.surgicalriskcalculator.com/miorcardiacarrest, www.riskcalculator.facs.org).

The Revised Cardiac Risk Index has six clinical predictors of major perioperative cardiac events:

  • History of cerebrovascular disease
  • Prior or current compensated congestive heart failure
  • History of coronary artery disease
  • Insulin-dependent diabetes mellitus
  • Renal insufficiency, defined as a serum creatinine level of 2 mg/dL or higher
  • Patient undergoing suprainguinal vascular, intraperitoneal, or intrathoracic surgery.

A patient who has 0 or 1 of these predictors would have a low risk of a major adverse cardiac event, whereas a patient with 2 or more would have elevated risk. These risk factors must be taken into consideration to determine the need, if any, for a preoperative ECG.

 

 

What an ECG can tell us

Abnormalities such as left ventricular hypertrophy, ST-segment depression, and pathologic Q waves on a preoperative ECG in a patient undergoing an elevated-risk surgical procedure may predict adverse perioperative cardiac events.3,6

In a retrospective study of 23,036 patients, Noordzij et al7 found that in patients undergoing elevated-risk surgery, those with an abnormal preoperative ECG had a higher incidence of cardiovascular death than those with a normal ECG. However, a preoperative ECG was obtained only in patients with established coronary artery disease or risk factors for cardiovascular disease. Hence, although an abnormal ECG in such patients undergoing elevated-risk surgery was predictive of adverse postoperative cardiac outcomes, we cannot say that the same would apply to patients without these characteristics undergoing elevated-risk surgery.

In a prospective observational study of patients with known coronary artery disease undergoing major noncardiac surgery, a preoperative ECG was found to contain prognostic information and was predictive of long-term outcome independent of clinical findings and perioperative ischemia.8

CURRENT GUIDELINES AND RECOMMENDATIONS

Several guidelines address whether to order a preoperative ECG but are mostly based on low-level evidence and expert opinion.1,2,6,9

Current guidelines recommend obtaining a preoperative ECG in patients with known coronary, peripheral arterial, or cerebrovascular disease.1,6,9

Obesity and associated comorbidities such as coronary artery disease, heart failure, systemic hypertension, and sleep apnea can predispose to increased perioperative complications. A preoperative 12-lead ECG is reasonable in morbidly obese patients (body mass index ≥ 40 kg/m2) and in obese patients (body mass index ≥ 30 kg/m2) with at least one risk factor for coronary artery disease or poor exercise tolerance, or both.10

Liu et al11 looked at the predictive value of a preoperative 12-lead ECG in 513 elderly patients (age ≥ 70) undergoing noncardiac surgery and found that electrocardiographic abnormalities were not predictive of adverse cardiac outcomes. In this study, although electrocardiographic abnormalities were common (noted in 75% of the patients), they were nonspecific and less useful in predicting postoperative cardiac complications than was the presence of comorbidities.11 Age alone as a cutoff for obtaining a preoperative ECG is not predictive of postoperative outcomes and a preoperative ECG is not warranted in all elderly patients. This is also reflected in current ACC/AHA guidelines on perioperative cardiovascular evaluation1 and is a change from prior ACC/AHA guidelines when age was used as a criterion for preoperative ECGs.12

Current guidelines do not recommend getting a preoperative ECG in asymptomatic patients undergoing low-cardiac-risk surgery.1,4,9

Although the ideal time for ordering an ECG before a planned surgery is unknown, obtaining one within 90 days before the surgery is considered adequate in stable patients in whom an ECG is indicated.1

BACK TO OUR PATIENT

On the basis of current evidence, our patient does not need a preoperative ECG, as it is unlikely to alter his perioperative management and instead may delay his surgery unnecessarily if any nonspecific changes prompt further cardiac workup.

CLINICAL BOTTOM LINE

Although frequently ordered in clinical practice, preoperative electrocardiography has a limited role in predicting postoperative outcome and should be ordered only in the appropriate clinical setting.1 Moreover, there is little evidence that outcomes are better if we obtain an ECG before surgery. The clinician should consider patient factors and the type of surgery before ordering diagnostic tests, including electrocardiography.

In asymptomatic patients undergoing nonemergent surgery:

  • It is reasonable to obtain a preoperative ECG in patients with known coronary artery disease, significant arrhythmia, peripheral arterial disease, cerebrovascular disease, chronic heart failure, or other significant structural heart disease undergoing elevated-cardiac-risk surgery.
  • Do not order a preoperative ECG in asymptomatic patients undergoing low-risk surgery.
  • Obtaining a preoperative ECG is reasonable in morbidly obese patients and in obese patients with one or more risk factors for coronary artery disease, or poor exercise tolerance, undergoing high-risk surgery.
References
  1. Fleisher LA, Fleischmann KE, Auerbach AD, et al. 2014 ACC/AHA guideline on perioperative cardiovascular evaluation and management of patients undergoing noncardiac surgery. J Am Coll Cardiol 2014; Jul 29. 10.1016/j.jacc.2014.07.944. [Epub ahead of print]
  2. Feely MA, Collins CS, Daniels PR, Kebede EB, Jatoi A, Mauck KF. Preoperative testing before noncardiac surgery: guidelines and recommendations. Am Fam Physician 2013; 87:414418.
  3. Lee TH, Marcantonio ER, Mangione CM, et al. Derivation and prospective validation of a simple index for prediction of cardiac risk of major noncardiac surgery. Circulation 1999; 100:10431049.
  4. Task Force for Preoperative Cardiac Risk Assessment and Perioperative Cardiac Management in Non-cardiac Surgery; European Society of Cardiology (ESC); Poldermans D, Bax JJ, Boersma E, et al. Guidelines for pre-operative cardiac risk assessment and perioperative cardiac management in non-cardiac surgery. Eur Heart J 2009; 30:27692812.
  5. Bilimoria KY, Liu Y, Paruch JL, Zhou L, Kmiecik TE, Ko CY, et al. Development and evaluation of the universal ACS NSQIP surgical risk calculator: a decision aid and informed consent tool for patients and surgeons. J Am Coll Surg 2013; 217:833842.
  6. Landesberg G, Einav S, Christopherson R, et al. Perioperative ischemia and cardiac complications in major vascular surgery: importance of the preoperative twelve-lead electrocardiogram. J Vasc Surg 1997; 26:570578.
  7. Noordzij PG, Boersma E, Bax JJ, et al. Prognostic value of routine preoperative electrocardiography in patients undergoing noncardiac surgery. Am J Cardiol 2006; 97:11031106.
  8. Jeger RV, Probst C, Arsenic R, et al. Long-term prognostic value of the preoperative 12-lead electrocardiogram before major noncardiac surgery in coronary artery disease. Am Heart J 2006; 151:508513.
  9. Committee on Standards and Practice Parameters; Apfelbaum JL, Connis RT, Nickinovich DG, Pasternak LR, Arens JF, Caplan RA, et al. Practice advisory for preanesthesia evaluation: an updated report by the American Society of Anesthesiologists Task Force on Preanesthesia Evaluation. Anesthesiology 2012; 116:522538.
  10. Poirier P, Alpert MA, Fleisher LA, et al. Cardiovascular evaluation and management of severely obese patients undergoing surgery: a science advisory from the American Heart Association. Circulation 2009; 120:8695.
  11. Liu LL, Dzankic S, Leung JM. Preoperative electrocardiogram abnormalities do not predict postoperative cardiac complications in geriatric surgical patients. J Am Geriatr Soc 2002; 50:11861191.
  12. Eagle KA, Berger PB, Calkins H, et al; American College of Cardiology; American Heart Association. ACC/AHA guideline update for perioperative cardiovascular evaluation for noncardiac surgery—executive summary. J Am Coll Cardiol 2002; 39:542553.
Article PDF
Author and Disclosure Information

Prashant Sharma, MD, FACP
Assistant Professor of Medicine, Department of Hospital Internal Medicine, Mayo Clinic, Rochester, MN

Sourab Dhungel, MD, FACP
Assistant Professor of Clinical Medicine, Section of Hospital Medicine, Temple University Hospital, Philadelphia, PA

Anbazhagan Prabhakaran, MD, MRCP (Edin), FACP
Assistant Clinical Professor of Medicine, Cleveland Clinic Lerner College of Medicine of Case Western Reserve University, Cleveland, OH; Medical Director, IMPACT Center, Cleveland Clinic

Address: Prashant Sharma, MD, FACP, Department of Hospital Internal Medicine, Mayo Clinic, 200 1st Street SW, OL-4, Rochester, MN 55905; e-mail: sharma.prashant@mayo.edu

Smart Testing is a joint project of the Cleveland Clinic Journal of Medicine and the American College of Physicians (ACP). The series, an extension of the ACP High Value Care initiative (hvc.acponline.org/index.html), provides recommendations for improving patient outcomes while reducing unnecessary testing and treatment.

Issue
Cleveland Clinic Journal of Medicine - 81(10)
Publications
Topics
Page Number
594-596
Sections
Author and Disclosure Information

Prashant Sharma, MD, FACP
Assistant Professor of Medicine, Department of Hospital Internal Medicine, Mayo Clinic, Rochester, MN

Sourab Dhungel, MD, FACP
Assistant Professor of Clinical Medicine, Section of Hospital Medicine, Temple University Hospital, Philadelphia, PA

Anbazhagan Prabhakaran, MD, MRCP (Edin), FACP
Assistant Clinical Professor of Medicine, Cleveland Clinic Lerner College of Medicine of Case Western Reserve University, Cleveland, OH; Medical Director, IMPACT Center, Cleveland Clinic

Address: Prashant Sharma, MD, FACP, Department of Hospital Internal Medicine, Mayo Clinic, 200 1st Street SW, OL-4, Rochester, MN 55905; e-mail: sharma.prashant@mayo.edu

Smart Testing is a joint project of the Cleveland Clinic Journal of Medicine and the American College of Physicians (ACP). The series, an extension of the ACP High Value Care initiative (hvc.acponline.org/index.html), provides recommendations for improving patient outcomes while reducing unnecessary testing and treatment.

Author and Disclosure Information

Prashant Sharma, MD, FACP
Assistant Professor of Medicine, Department of Hospital Internal Medicine, Mayo Clinic, Rochester, MN

Sourab Dhungel, MD, FACP
Assistant Professor of Clinical Medicine, Section of Hospital Medicine, Temple University Hospital, Philadelphia, PA

Anbazhagan Prabhakaran, MD, MRCP (Edin), FACP
Assistant Clinical Professor of Medicine, Cleveland Clinic Lerner College of Medicine of Case Western Reserve University, Cleveland, OH; Medical Director, IMPACT Center, Cleveland Clinic

Address: Prashant Sharma, MD, FACP, Department of Hospital Internal Medicine, Mayo Clinic, 200 1st Street SW, OL-4, Rochester, MN 55905; e-mail: sharma.prashant@mayo.edu

Smart Testing is a joint project of the Cleveland Clinic Journal of Medicine and the American College of Physicians (ACP). The series, an extension of the ACP High Value Care initiative (hvc.acponline.org/index.html), provides recommendations for improving patient outcomes while reducing unnecessary testing and treatment.

Article PDF
Article PDF

A 55-year-old lawyer with hypertension well controlled on lisinopril and amlodipine is scheduled for elective hernia repair under general anesthesia. His surgeon has referred him for a preoperative evaluation. He has never smoked, runs 4 miles on days off from work, and enjoys long hiking trips. On examination, his body mass index is 26 kg/m2 and his blood pressure is 130/78 mm Hg; his cardiac examination and the rest of the clinical examination are unremarkable. He asks if he should have an electrocardiogram (ECG) as a part of his workup.

A preoperative ECG is not routinely recommended in all asymptomatic patients undergoing noncardiac surgery.

Consider obtaining an ECG in patients planning to undergo a high-risk surgical procedure, especially if they have one or more clinical risk factors for coronary artery disease, and in patients undergoing elevated-cardiac-risk surgery who are known to have coronary artery disease, chronic heart failure, peripheral arterial disease, or cerebrovascular disease. However, a preoperative ECG is not routinely recommended for patients perceived to be at low cardiac risk who are planning to undergo low-risk surgery. In those patients it could delay surgery unnecessarily, cause further unnecessary testing, drive up costs, and increase patient anxiety.

Here we discuss the perioperative cardiac risk based on type of surgery and patient characteristics and summarize the current guidelines and recommendations on obtaining a preoperative 12-lead ECG in patients undergoing noncardiac surgery.

RISK DEPENDS ON TYPE OF SURGERY AND PATIENT FACTORS

Physicians, including primary care physicians, hospitalists, cardiologists, and anesthesiologists, are routinely asked to evaluate patients before surgical procedures. The purpose of the preoperative evaluation is to optimize existing medical conditions, to identify undiagnosed conditions that can increase risk of perioperative morbidity and death, and to suggest strategies to mitigate risk.1,2

Cardiac risk is multifactorial, and risk factors for postoperative adverse cardiac events include the type of surgery and patient factors.1,3

Cardiac risk based on type of surgery

Low-risk procedures are those in which the risk of a perioperative major adverse cardiac event is less than 1%.1,4 Examples:

  • Ambulatory surgery
  • Breast or plastic surgery
  • Cataract surgery
  • Endoscopic procedures.

Elevated-risk procedures are those in which the risk is 1% or higher. Examples:

  • Intraperitoneal surgery
  • Intrathoracic surgery
  • Carotid endarterectomy
  • Head and neck surgery
  • Orthopedic surgery
  • Prostate surgery
  • Aortic surgery
  • Major vascular surgery
  • Peripheral arterial surgery.

Cardiac risk based on patient factors

The 2014 American College of Cardiology and American Heart Association (ACC/AHA) perioperative guidelines list a number of clinical risk factors for perioperative cardiac morbidity and death.1 These include coronary artery disease, chronic heart failure, clinically suspected moderate or greater degrees of valvular heart disease, arrhythmias, conduction disorders, pulmonary vascular disease, and adult congenital heart disease.

Patients with these conditions and patients with unstable coronary syndromes warrant preoperative ECGs and sometimes even urgent interventions before any nonemergency surgery, provided such interventions would affect decision-making and perioperative care.1

The risk of perioperative cardiac morbidity and death can be calculated using either the Revised Cardiac Risk Index scoring system or the American College of Surgeons National Surgical Quality Improvement Program calculator.157 The former is fairly simple, validated, and accepted, while the latter requires use of online calculators (eg, www.surgicalriskcalculator.com/miorcardiacarrest, www.riskcalculator.facs.org).

The Revised Cardiac Risk Index has six clinical predictors of major perioperative cardiac events:

  • History of cerebrovascular disease
  • Prior or current compensated congestive heart failure
  • History of coronary artery disease
  • Insulin-dependent diabetes mellitus
  • Renal insufficiency, defined as a serum creatinine level of 2 mg/dL or higher
  • Patient undergoing suprainguinal vascular, intraperitoneal, or intrathoracic surgery.

A patient who has 0 or 1 of these predictors would have a low risk of a major adverse cardiac event, whereas a patient with 2 or more would have elevated risk. These risk factors must be taken into consideration to determine the need, if any, for a preoperative ECG.

 

 

What an ECG can tell us

Abnormalities such as left ventricular hypertrophy, ST-segment depression, and pathologic Q waves on a preoperative ECG in a patient undergoing an elevated-risk surgical procedure may predict adverse perioperative cardiac events.3,6

In a retrospective study of 23,036 patients, Noordzij et al7 found that in patients undergoing elevated-risk surgery, those with an abnormal preoperative ECG had a higher incidence of cardiovascular death than those with a normal ECG. However, a preoperative ECG was obtained only in patients with established coronary artery disease or risk factors for cardiovascular disease. Hence, although an abnormal ECG in such patients undergoing elevated-risk surgery was predictive of adverse postoperative cardiac outcomes, we cannot say that the same would apply to patients without these characteristics undergoing elevated-risk surgery.

In a prospective observational study of patients with known coronary artery disease undergoing major noncardiac surgery, a preoperative ECG was found to contain prognostic information and was predictive of long-term outcome independent of clinical findings and perioperative ischemia.8

CURRENT GUIDELINES AND RECOMMENDATIONS

Several guidelines address whether to order a preoperative ECG but are mostly based on low-level evidence and expert opinion.1,2,6,9

Current guidelines recommend obtaining a preoperative ECG in patients with known coronary, peripheral arterial, or cerebrovascular disease.1,6,9

Obesity and associated comorbidities such as coronary artery disease, heart failure, systemic hypertension, and sleep apnea can predispose to increased perioperative complications. A preoperative 12-lead ECG is reasonable in morbidly obese patients (body mass index ≥ 40 kg/m2) and in obese patients (body mass index ≥ 30 kg/m2) with at least one risk factor for coronary artery disease or poor exercise tolerance, or both.10

Liu et al11 looked at the predictive value of a preoperative 12-lead ECG in 513 elderly patients (age ≥ 70) undergoing noncardiac surgery and found that electrocardiographic abnormalities were not predictive of adverse cardiac outcomes. In this study, although electrocardiographic abnormalities were common (noted in 75% of the patients), they were nonspecific and less useful in predicting postoperative cardiac complications than was the presence of comorbidities.11 Age alone as a cutoff for obtaining a preoperative ECG is not predictive of postoperative outcomes and a preoperative ECG is not warranted in all elderly patients. This is also reflected in current ACC/AHA guidelines on perioperative cardiovascular evaluation1 and is a change from prior ACC/AHA guidelines when age was used as a criterion for preoperative ECGs.12

Current guidelines do not recommend getting a preoperative ECG in asymptomatic patients undergoing low-cardiac-risk surgery.1,4,9

Although the ideal time for ordering an ECG before a planned surgery is unknown, obtaining one within 90 days before the surgery is considered adequate in stable patients in whom an ECG is indicated.1

BACK TO OUR PATIENT

On the basis of current evidence, our patient does not need a preoperative ECG, as it is unlikely to alter his perioperative management and instead may delay his surgery unnecessarily if any nonspecific changes prompt further cardiac workup.

CLINICAL BOTTOM LINE

Although frequently ordered in clinical practice, preoperative electrocardiography has a limited role in predicting postoperative outcome and should be ordered only in the appropriate clinical setting.1 Moreover, there is little evidence that outcomes are better if we obtain an ECG before surgery. The clinician should consider patient factors and the type of surgery before ordering diagnostic tests, including electrocardiography.

In asymptomatic patients undergoing nonemergent surgery:

  • It is reasonable to obtain a preoperative ECG in patients with known coronary artery disease, significant arrhythmia, peripheral arterial disease, cerebrovascular disease, chronic heart failure, or other significant structural heart disease undergoing elevated-cardiac-risk surgery.
  • Do not order a preoperative ECG in asymptomatic patients undergoing low-risk surgery.
  • Obtaining a preoperative ECG is reasonable in morbidly obese patients and in obese patients with one or more risk factors for coronary artery disease, or poor exercise tolerance, undergoing high-risk surgery.

A 55-year-old lawyer with hypertension well controlled on lisinopril and amlodipine is scheduled for elective hernia repair under general anesthesia. His surgeon has referred him for a preoperative evaluation. He has never smoked, runs 4 miles on days off from work, and enjoys long hiking trips. On examination, his body mass index is 26 kg/m2 and his blood pressure is 130/78 mm Hg; his cardiac examination and the rest of the clinical examination are unremarkable. He asks if he should have an electrocardiogram (ECG) as a part of his workup.

A preoperative ECG is not routinely recommended in all asymptomatic patients undergoing noncardiac surgery.

Consider obtaining an ECG in patients planning to undergo a high-risk surgical procedure, especially if they have one or more clinical risk factors for coronary artery disease, and in patients undergoing elevated-cardiac-risk surgery who are known to have coronary artery disease, chronic heart failure, peripheral arterial disease, or cerebrovascular disease. However, a preoperative ECG is not routinely recommended for patients perceived to be at low cardiac risk who are planning to undergo low-risk surgery. In those patients it could delay surgery unnecessarily, cause further unnecessary testing, drive up costs, and increase patient anxiety.

Here we discuss the perioperative cardiac risk based on type of surgery and patient characteristics and summarize the current guidelines and recommendations on obtaining a preoperative 12-lead ECG in patients undergoing noncardiac surgery.

RISK DEPENDS ON TYPE OF SURGERY AND PATIENT FACTORS

Physicians, including primary care physicians, hospitalists, cardiologists, and anesthesiologists, are routinely asked to evaluate patients before surgical procedures. The purpose of the preoperative evaluation is to optimize existing medical conditions, to identify undiagnosed conditions that can increase risk of perioperative morbidity and death, and to suggest strategies to mitigate risk.1,2

Cardiac risk is multifactorial, and risk factors for postoperative adverse cardiac events include the type of surgery and patient factors.1,3

Cardiac risk based on type of surgery

Low-risk procedures are those in which the risk of a perioperative major adverse cardiac event is less than 1%.1,4 Examples:

  • Ambulatory surgery
  • Breast or plastic surgery
  • Cataract surgery
  • Endoscopic procedures.

Elevated-risk procedures are those in which the risk is 1% or higher. Examples:

  • Intraperitoneal surgery
  • Intrathoracic surgery
  • Carotid endarterectomy
  • Head and neck surgery
  • Orthopedic surgery
  • Prostate surgery
  • Aortic surgery
  • Major vascular surgery
  • Peripheral arterial surgery.

Cardiac risk based on patient factors

The 2014 American College of Cardiology and American Heart Association (ACC/AHA) perioperative guidelines list a number of clinical risk factors for perioperative cardiac morbidity and death.1 These include coronary artery disease, chronic heart failure, clinically suspected moderate or greater degrees of valvular heart disease, arrhythmias, conduction disorders, pulmonary vascular disease, and adult congenital heart disease.

Patients with these conditions and patients with unstable coronary syndromes warrant preoperative ECGs and sometimes even urgent interventions before any nonemergency surgery, provided such interventions would affect decision-making and perioperative care.1

The risk of perioperative cardiac morbidity and death can be calculated using either the Revised Cardiac Risk Index scoring system or the American College of Surgeons National Surgical Quality Improvement Program calculator.157 The former is fairly simple, validated, and accepted, while the latter requires use of online calculators (eg, www.surgicalriskcalculator.com/miorcardiacarrest, www.riskcalculator.facs.org).

The Revised Cardiac Risk Index has six clinical predictors of major perioperative cardiac events:

  • History of cerebrovascular disease
  • Prior or current compensated congestive heart failure
  • History of coronary artery disease
  • Insulin-dependent diabetes mellitus
  • Renal insufficiency, defined as a serum creatinine level of 2 mg/dL or higher
  • Patient undergoing suprainguinal vascular, intraperitoneal, or intrathoracic surgery.

A patient who has 0 or 1 of these predictors would have a low risk of a major adverse cardiac event, whereas a patient with 2 or more would have elevated risk. These risk factors must be taken into consideration to determine the need, if any, for a preoperative ECG.

 

 

What an ECG can tell us

Abnormalities such as left ventricular hypertrophy, ST-segment depression, and pathologic Q waves on a preoperative ECG in a patient undergoing an elevated-risk surgical procedure may predict adverse perioperative cardiac events.3,6

In a retrospective study of 23,036 patients, Noordzij et al7 found that in patients undergoing elevated-risk surgery, those with an abnormal preoperative ECG had a higher incidence of cardiovascular death than those with a normal ECG. However, a preoperative ECG was obtained only in patients with established coronary artery disease or risk factors for cardiovascular disease. Hence, although an abnormal ECG in such patients undergoing elevated-risk surgery was predictive of adverse postoperative cardiac outcomes, we cannot say that the same would apply to patients without these characteristics undergoing elevated-risk surgery.

In a prospective observational study of patients with known coronary artery disease undergoing major noncardiac surgery, a preoperative ECG was found to contain prognostic information and was predictive of long-term outcome independent of clinical findings and perioperative ischemia.8

CURRENT GUIDELINES AND RECOMMENDATIONS

Several guidelines address whether to order a preoperative ECG but are mostly based on low-level evidence and expert opinion.1,2,6,9

Current guidelines recommend obtaining a preoperative ECG in patients with known coronary, peripheral arterial, or cerebrovascular disease.1,6,9

Obesity and associated comorbidities such as coronary artery disease, heart failure, systemic hypertension, and sleep apnea can predispose to increased perioperative complications. A preoperative 12-lead ECG is reasonable in morbidly obese patients (body mass index ≥ 40 kg/m2) and in obese patients (body mass index ≥ 30 kg/m2) with at least one risk factor for coronary artery disease or poor exercise tolerance, or both.10

Liu et al11 looked at the predictive value of a preoperative 12-lead ECG in 513 elderly patients (age ≥ 70) undergoing noncardiac surgery and found that electrocardiographic abnormalities were not predictive of adverse cardiac outcomes. In this study, although electrocardiographic abnormalities were common (noted in 75% of the patients), they were nonspecific and less useful in predicting postoperative cardiac complications than was the presence of comorbidities.11 Age alone as a cutoff for obtaining a preoperative ECG is not predictive of postoperative outcomes and a preoperative ECG is not warranted in all elderly patients. This is also reflected in current ACC/AHA guidelines on perioperative cardiovascular evaluation1 and is a change from prior ACC/AHA guidelines when age was used as a criterion for preoperative ECGs.12

Current guidelines do not recommend getting a preoperative ECG in asymptomatic patients undergoing low-cardiac-risk surgery.1,4,9

Although the ideal time for ordering an ECG before a planned surgery is unknown, obtaining one within 90 days before the surgery is considered adequate in stable patients in whom an ECG is indicated.1

BACK TO OUR PATIENT

On the basis of current evidence, our patient does not need a preoperative ECG, as it is unlikely to alter his perioperative management and instead may delay his surgery unnecessarily if any nonspecific changes prompt further cardiac workup.

CLINICAL BOTTOM LINE

Although frequently ordered in clinical practice, preoperative electrocardiography has a limited role in predicting postoperative outcome and should be ordered only in the appropriate clinical setting.1 Moreover, there is little evidence that outcomes are better if we obtain an ECG before surgery. The clinician should consider patient factors and the type of surgery before ordering diagnostic tests, including electrocardiography.

In asymptomatic patients undergoing nonemergent surgery:

  • It is reasonable to obtain a preoperative ECG in patients with known coronary artery disease, significant arrhythmia, peripheral arterial disease, cerebrovascular disease, chronic heart failure, or other significant structural heart disease undergoing elevated-cardiac-risk surgery.
  • Do not order a preoperative ECG in asymptomatic patients undergoing low-risk surgery.
  • Obtaining a preoperative ECG is reasonable in morbidly obese patients and in obese patients with one or more risk factors for coronary artery disease, or poor exercise tolerance, undergoing high-risk surgery.
References
  1. Fleisher LA, Fleischmann KE, Auerbach AD, et al. 2014 ACC/AHA guideline on perioperative cardiovascular evaluation and management of patients undergoing noncardiac surgery. J Am Coll Cardiol 2014; Jul 29. 10.1016/j.jacc.2014.07.944. [Epub ahead of print]
  2. Feely MA, Collins CS, Daniels PR, Kebede EB, Jatoi A, Mauck KF. Preoperative testing before noncardiac surgery: guidelines and recommendations. Am Fam Physician 2013; 87:414418.
  3. Lee TH, Marcantonio ER, Mangione CM, et al. Derivation and prospective validation of a simple index for prediction of cardiac risk of major noncardiac surgery. Circulation 1999; 100:10431049.
  4. Task Force for Preoperative Cardiac Risk Assessment and Perioperative Cardiac Management in Non-cardiac Surgery; European Society of Cardiology (ESC); Poldermans D, Bax JJ, Boersma E, et al. Guidelines for pre-operative cardiac risk assessment and perioperative cardiac management in non-cardiac surgery. Eur Heart J 2009; 30:27692812.
  5. Bilimoria KY, Liu Y, Paruch JL, Zhou L, Kmiecik TE, Ko CY, et al. Development and evaluation of the universal ACS NSQIP surgical risk calculator: a decision aid and informed consent tool for patients and surgeons. J Am Coll Surg 2013; 217:833842.
  6. Landesberg G, Einav S, Christopherson R, et al. Perioperative ischemia and cardiac complications in major vascular surgery: importance of the preoperative twelve-lead electrocardiogram. J Vasc Surg 1997; 26:570578.
  7. Noordzij PG, Boersma E, Bax JJ, et al. Prognostic value of routine preoperative electrocardiography in patients undergoing noncardiac surgery. Am J Cardiol 2006; 97:11031106.
  8. Jeger RV, Probst C, Arsenic R, et al. Long-term prognostic value of the preoperative 12-lead electrocardiogram before major noncardiac surgery in coronary artery disease. Am Heart J 2006; 151:508513.
  9. Committee on Standards and Practice Parameters; Apfelbaum JL, Connis RT, Nickinovich DG, Pasternak LR, Arens JF, Caplan RA, et al. Practice advisory for preanesthesia evaluation: an updated report by the American Society of Anesthesiologists Task Force on Preanesthesia Evaluation. Anesthesiology 2012; 116:522538.
  10. Poirier P, Alpert MA, Fleisher LA, et al. Cardiovascular evaluation and management of severely obese patients undergoing surgery: a science advisory from the American Heart Association. Circulation 2009; 120:8695.
  11. Liu LL, Dzankic S, Leung JM. Preoperative electrocardiogram abnormalities do not predict postoperative cardiac complications in geriatric surgical patients. J Am Geriatr Soc 2002; 50:11861191.
  12. Eagle KA, Berger PB, Calkins H, et al; American College of Cardiology; American Heart Association. ACC/AHA guideline update for perioperative cardiovascular evaluation for noncardiac surgery—executive summary. J Am Coll Cardiol 2002; 39:542553.
References
  1. Fleisher LA, Fleischmann KE, Auerbach AD, et al. 2014 ACC/AHA guideline on perioperative cardiovascular evaluation and management of patients undergoing noncardiac surgery. J Am Coll Cardiol 2014; Jul 29. 10.1016/j.jacc.2014.07.944. [Epub ahead of print]
  2. Feely MA, Collins CS, Daniels PR, Kebede EB, Jatoi A, Mauck KF. Preoperative testing before noncardiac surgery: guidelines and recommendations. Am Fam Physician 2013; 87:414418.
  3. Lee TH, Marcantonio ER, Mangione CM, et al. Derivation and prospective validation of a simple index for prediction of cardiac risk of major noncardiac surgery. Circulation 1999; 100:10431049.
  4. Task Force for Preoperative Cardiac Risk Assessment and Perioperative Cardiac Management in Non-cardiac Surgery; European Society of Cardiology (ESC); Poldermans D, Bax JJ, Boersma E, et al. Guidelines for pre-operative cardiac risk assessment and perioperative cardiac management in non-cardiac surgery. Eur Heart J 2009; 30:27692812.
  5. Bilimoria KY, Liu Y, Paruch JL, Zhou L, Kmiecik TE, Ko CY, et al. Development and evaluation of the universal ACS NSQIP surgical risk calculator: a decision aid and informed consent tool for patients and surgeons. J Am Coll Surg 2013; 217:833842.
  6. Landesberg G, Einav S, Christopherson R, et al. Perioperative ischemia and cardiac complications in major vascular surgery: importance of the preoperative twelve-lead electrocardiogram. J Vasc Surg 1997; 26:570578.
  7. Noordzij PG, Boersma E, Bax JJ, et al. Prognostic value of routine preoperative electrocardiography in patients undergoing noncardiac surgery. Am J Cardiol 2006; 97:11031106.
  8. Jeger RV, Probst C, Arsenic R, et al. Long-term prognostic value of the preoperative 12-lead electrocardiogram before major noncardiac surgery in coronary artery disease. Am Heart J 2006; 151:508513.
  9. Committee on Standards and Practice Parameters; Apfelbaum JL, Connis RT, Nickinovich DG, Pasternak LR, Arens JF, Caplan RA, et al. Practice advisory for preanesthesia evaluation: an updated report by the American Society of Anesthesiologists Task Force on Preanesthesia Evaluation. Anesthesiology 2012; 116:522538.
  10. Poirier P, Alpert MA, Fleisher LA, et al. Cardiovascular evaluation and management of severely obese patients undergoing surgery: a science advisory from the American Heart Association. Circulation 2009; 120:8695.
  11. Liu LL, Dzankic S, Leung JM. Preoperative electrocardiogram abnormalities do not predict postoperative cardiac complications in geriatric surgical patients. J Am Geriatr Soc 2002; 50:11861191.
  12. Eagle KA, Berger PB, Calkins H, et al; American College of Cardiology; American Heart Association. ACC/AHA guideline update for perioperative cardiovascular evaluation for noncardiac surgery—executive summary. J Am Coll Cardiol 2002; 39:542553.
Issue
Cleveland Clinic Journal of Medicine - 81(10)
Issue
Cleveland Clinic Journal of Medicine - 81(10)
Page Number
594-596
Page Number
594-596
Publications
Publications
Topics
Article Type
Display Headline
Should all patients have a resting 12-lead ECG before elective noncardiac surgery?
Display Headline
Should all patients have a resting 12-lead ECG before elective noncardiac surgery?
Sections
Disallow All Ads
Alternative CME
Article PDF Media